You are on page 1of 73

Shoulder and Elbow Scored and Recorded Self-

Assessment Examination 2019


Question 1 of 100
Figures 1 through 3 are the radiographs of a 55-year-old woman who fell on her outstretched right arm,
resulting in acute elbow pain and swelling. On examination, she has lateral elbow bruising with mechanical
block to supination and pronation. She has no medial tenderness. She is unable to extend her elbow within 60°
of full extension. During surgery utilizing a direct lateral approach, the surgeon observes a completely bare
lateral epicondyle. After surgical repair, a stable and congruent joint is achieved. Initial postoperative
rehabilitation should include

A. 3 weeks of cast immobilization.


B. elbow extension exercises with the forearm supinated.
C. elbow extension exercises with the forearm pronated.
D. elbow extension exercises with the forearm in neutral rotation.
PREFERRED RESPONSE: C
DISCUSSION:
Radial head fractures are thought to occur as a result of valgus posterolateral rotary load across the elbow,
although the mechanism can certainly vary. Minimally or nondisplaced fractures without any clinical
instability or block to motion can often be successfully managed non-surgically. Fractures with >2 mm of
displacement or fragments that block motion require surgical repair. A critical aspect during surgery is
identifying concomitant injury to the lateral collateral ligament complex (LCL). When encountered, the LCL
will be avulsed from its origin from the lateral epicondyle, resulting in a bare area. After the radial head is
either reduced and fixed or replaced (Figures 4 and 5), the LCL should be repaired back to its anatomic origin.
Postoperatively, the surgeon must communicate to the therapist that elbow extension exercises should be
performed with the forearm in pronation as a result of the compromised LCL. Elbow extension exercises in
supination and neutral are recommended for compromised medial collateral ligament or combined medial and
lateral ligament injury, respectively. Without any medial elbow bruising, swelling, or tenderness, it is unlikely
that the patient has an injury to the medial collateral ligament.

Question 2 of 100
A 75-year-old woman with rheumatoid arthritis and a long history of oral corticosteroid use sustains a
comminuted intra-articular distal humerus fracture. What is the best surgical option?
A. Open reduction internal fixation (ORIF) with parallel plates
B. ORIF with orthogonal plates and iliac crest bone grafting
C. Total elbow arthroplasty (TEA)
D. Closed reduction and percutaneous pinning

PREFERRED RESPONSE: C
DISCUSSION:
TEA is the best surgical option. McKee and associates published a multicenter randomized controlled trial
comparing ORIF with TEA in elderly patients. TEA resulted in better 2-year clinical functional scores and
more predictable outcomes compared with ORIF. TEA was also likely to result in a lower resurgical rate; one-
quarter of patients with fractures randomized to ORIF could not achieve stable fixation. Further, Frankle and
associates reported a comparative study of TEA versus ORIF in 24 elderly women. TEA outcomes were again
superior to ORIF at a minimum of 2 years of follow-up. TEA was especially useful in patients with
comorbidities that compromise bone stock, including osteoporosis and oral corticosteroid use. Closed
reduction and percutaneous pinning studies have not been published on the adult population.

Question 3 of 100
A complication associated with using the Morrey approach (triceps reflecting) to implant a semiconstrained
total elbow arthroplasty is
A. loss of elbow extensor power.
B. implant dislocation.
C. implant malposition.
D. development of heterotopic ossification.

PREFERRED RESPONSE: A
DISCUSSION:
Numerous approaches can be used to implant a total elbow arthroplasty. The Morrey approach identifies,
transposes, and protects the ulnar nerve, and then subperiosteally reflects the triceps off the ulna. The sleeve
of tissue is very thin distally, and the triceps need to be meticulously repaired at the time of closure. Implant
dislocation and malposition are less likely with an extensile approach, and dislocation is unlikely with a
semiconstrained implant. The development of heterotopic ossification is unrelated to the surgical approach
used for elbow arthroplasty.

Question 4 of 100
Figures 1 through 4 are the radiographs of a 55-year-old healthy woman who fell down a flight of steps while
sleepwalking. When the surgeon replaces the radial head, the elbow dislocates posteriorly at 60° of flexion as
it is brought out from full flexion. What is the best next step?

A. Only repair the lateral collateral ligament (LCL).


B. Do nothing further and place the elbow in 90° of flexion.
C. Repair the posterior band of the medial collateral ligament (MCL).
D. Repair the coronoid and reassess for stability.
PREFERRED RESPONSE: D
DISCUSSION:
The coronoid is important for elbow stability, particularly as the elbow is moved into extension. Repairing the
LCL alone after radial head replacement in “terrible triad” injuries may suffice when there is a type 1 coronoid
fracture or an anterior capsular avulsion. For more extensive coronoid injuries, live dynamic examination of
stability is needed to determine whether repair of the coronoid is needed. For this patient, doing nothing further
will lead to immediate postsurgical instability, and repairing the LCL complex alone will not lead to stability.
The posterior band of the MCL will not add to stability. The next step to attain stability is to repair the coronoid
fracture and re-examine the elbow for stability.

Question 5 of 100
A 45-year-old man falls from a skateboard and dislocates his elbow. After a closed reduction in the emergency
department, his elbow is carefully examined. He has positive valgus stress, moving valgus stress, and milking
maneuver tests. His elbow appears stable to varus stress and lateral pivot shift tests. What is the most
appropriate manner of immobilizing the elbow for this patient?
A. Sling for 3 days, with early active range of motion
B. Posterior splint for 5 to 7 days, forearm in full pronation
C. Posterior splint for 5 to 7 days, forearm in neutral
D. Posterior splint for 5 to 7 days, forearm in full supination

PREFERRED RESPONSE: D
DISCUSSION:
Varus posteromedial rotatory instability occurs following a fall onto an outstretched hand with axial loading
and a varus stress to the elbow. This injury can result in a rupture of the posterior band of the medial collateral
ligament (MCL), fracture of the anteromedial facet of the coronoid, and avulsion of the lateral ulnar collateral
ligament (LUCL). Based on the examination findings, this patient has an acute MCL rupture. Furthermore,
the LUCL appears intact, as evidence by the stability with varus stress. To protect the reduction in the acute
setting, posterior splinting is recommended, but placing the forearm in full supination tightens the structures
medially where the MCL is deficient. Splinting in neutral is indicated for valgus posterolateral rotatory
instability, where both the LUCL and MCL are ruptured. Splinting in full pronation is indicated for isolated
LUCL ruptures. Early active range of motion is not recommended for adults immediately after an acute elbow
dislocation, as ligamentous injury or fracture nearly always accompanies the dislocation. In this case, the
forearm should be splinted in full supination.

Question 6 of 100
A 38-year-old man sustains a terrible triad injury consisting of an elbow dislocation, comminuted and
displaced radial head fracture, and a type I coronoid fracture. Intraoperative findings after radial head
replacement and lateral collateral ligament complex repair reveal persistent instability consisting of medial
opening on valgus stress and posteromedial subluxation of the ulnohumeral and radiocapitellar joints. What
is the best next step?
A. Medial collateral ligament repair or reconstruction
B. Reconstruction of the radial collateral ligament
C. Resection of the type I coronoid fracture and capsular repair to the remaining coronoid
D. Open reduction and buttress plating of the coronoid fracture

PREFERRED RESPONSE: A
DISCUSSION:
Terrible triad injuries of the elbow are common, and the management of type I coronoid tip fractures remains
controversial. Type I coronoid fractures result in only small changes in elbow kinematics that have been shown
to be uncorrected with suture repair. A type I coronoid tip fracture is not amenable to buttress plate fixation.
The radial collateral ligament is a component of the lateral collateral ligament complex and has already been
repaired. The persistent medial laxity and posteromedial joint subluxation noted is indicative of ongoing
instability. The next step would be repair or reconstruction of the medial collateral ligament, which will
normally correct the medial instability. Articulated versus static external fixation can be considered if
restoration of the ligamentous constraint of the medial side of the elbow cannot be accomplished surgically.

Question 7 of 100
When performing an ulnar nerve decompression at the elbow, the surgeon must be aware of the

A. median nerve as it crosses the surgical field 6 cm proximal to the medial epicondyle.
B. medial antebrachial cutaneous nerve as it crosses the field 3 cm distal to the medial epicondyle.
C. anterior antebrachial cutaneous nerve as it crosses the field at the medial epicondyle.
D. posterior antebrachial cutaneous nerve that crosses the field 2 cm distal to the medial epicondyle.
PREFERRED RESPONSE: B
DISCUSSION:
The medial antebrachial cutaneous and medial brachial cutaneous are nerves that can be injured during ulnar
nerve decompression at the elbow. The medial antebrachial cutaneous nerve crosses the surgical field at an
average of 3.1 cm distal to the medial epicondyle. The medial brachial cutaneous nerve crosses the field 7 cm
proximal to the medial epicondyle and arborizes into two to three terminal branches. Because the surgical
approach involves dissection on the medial side, the posterior antebrachial cutaneous nerve is distant from the
exposure. Although the median nerve potentially can be located in the deep dissection of a submuscular
transposition, it is considered distant to an in situ decompression.

Question 8 of 100
Figures 1 through 3 are the radiographs of a 45-year-old man following acute trauma.

Following radial head replacement, the elbow exhibits persistent laxity to valgus stress in extension. What is
the best next step to regain stability?

A. Posterior capsular repair


B. Anterior capsular repair
C. Fixation of the type I coronoid fracture
D. Repair of the medial collateral ligament (MCL)
PREFERRED RESPONSE: D
DISCUSSION:
Longitudinal forearm instability is an indication for radial head replacement to prevent proximal migration of
the radial shaft. Radial head replacement is indicated in radial head fractures involving three or more
fragments. Younger age is not a contraindication or indication for radial head replacement. Anteromedial
coronoid facet fractures usually are associated with a posteromedial rotatory mechanism that does not increase
or decrease risk for radial head fractures necessitating replacement. Following restoration of the radial head,
a lateral collateral ligament repair would be the next step to restore stability if necessary. MCL repair would
restore medial stability if stability persisted following restoration of posterolateral laxity. Repair of type I
coronoid fractures does not substantially affect stability. Application of a hinged external fixator can restore
stability in severe cases but is usually reserved for refractory instability after ligament repair has been
performed and instability persists. Anterior and posterior capsular repair do not significantly affect instability.
MCL repair is generally the next step to obtain stability, with application of a hinged external fixator as a last
step to maintain joint congruity.

Question 9 of 100
A 42-year-old woman sustains a closed posterior elbow dislocation. A closed reduction is performed, and the
elbow appears stable under fluoroscopic examination. Initial treatment should consist of
A. early mobilization only.
B. surgical reconstruction of medial and lateral collateral ligaments.
C. active motion in a hinged brace from 30° to 120°.
D. application of hinged external fixator with early mobilization.

PREFERRED RESPONSE: A
DISCUSSION:
This is a simple (no associated fracture) elbow dislocation. Such dislocations can be treated with closed
reduction followed by mobilization after 5 to 7 days to avoid stiffness, provided the elbow is stable through a
full arc of motion at the time of reduction. If the elbow is unstable but has a short arc of stability, then using
a hinged brace in the stable arc may be considered. (Note: It may be necessary to splint the elbow in pronation
if the medial collateral ligament [MCL] is intact and the lateral collateral ligament [LCL] is disrupted, or in
supination if the LCL is intact but the MCL disrupted.) Surgical reconstruction of the LCL and MCL may be
required only if the elbow does not have a stable arc at the time of reduction. If unstable after reconstruction,
application of a hinged external fixator may be considered.

Question 10 of 100
Figures 1 through 3 are the radiographs of a 45-year-old man following acute trauma. Which radiographic
finding indicates the likely need for a radial head replacement?

A. Two or fewer fragments of the radial head


B. Age <21 years
C. Wrist pain and asymmetry of the ipsilateral distal radioulnar joint
D. Anteromedial coronoid comminution
PREFERRED RESPONSE: C
DISCUSSION:

Longitudinal forearm instability is an indication for radial head replacement to prevent proximal
migration of the radial shaft. Radial head replacement is indicated in radial head fractures involving three
or more fragments. Younger age is not a contraindication or indication for radial head replacement.
Anteromedial coronoid facet fractures usually are associated with a posteromedial rotatory mechanism
that does not increase or decrease risk for radial head fractures necessitating replacement. Following
restoration of the radial head, a lateral collateral ligament repair would be the next step to restore stability
if necessary. MCL repair would restore medial stability if stability persisted following restoration of
posterolateral laxity. Repair of type I coronoid fractures does not substantially affect stability. Application
of a hinged external fixator can restore stability in severe cases but is usually reserved for refractory
instability after ligament repair has been performed and instability persists. Anterior and posterior
capsular repair do not significantly affect instability. MCL repair is generally the next step to obtain
stability, with application of a hinged external fixator as a last step to maintain joint congruity.

Question 11 of 100
A 36-year-old right-hand-dominant man falls from his motorcycle and sustains the acute right upper extremity
injury seen in Figure 1. At surgery, an open reduction and internal fixation (ORIF) of the ulna is performed
along with attempted open reduction of the radiocapitellar joint. However, the radial head is slightly subluxed
in flexion and redislocates with elbow extension <90°. What is the most appropriate treatment at this time?

A. Radial head resection


B. Casting in 90° of flexion for 3 weeks, followed by reassessment of elbow stability
C. Repair of the annular ligament
D. Revision ORIF of the ulnar fracture
PREFERRED RESPONSE: C
DISCUSSION:
This case is a variant of a type I Monteggia fracture according to the Bado classification, with a segmental
ulna fracture. In some cases, the radial head subluxation can be subtle, and missing this would lead to a poor
outcome. In this case, the anterior radial head dislocation is obvious, but the segmental nature of the ulna
fracture makes anatomic reduction difficult. The radial head usually spontaneously reduces once the ulna is
anatomically reduced, and no surgical treatment to the lateral side is required. When this is not the case, a
lateral approach and incision of the annular ligament may be required for reduction. If an open reduction of
the radial head is unsuccessful, the problem is almost always residual malalignment of the ulna. Therefore,
casting and annular ligament repair will not improve reduction. A radial head resection would eliminate the
nonconcentric contact between radial head and capitellum but would not be an appropriate treatment for this
young patient who has an acute, correctable fracture deformity.

Question 12 of 100
A right-hand-dominant 45-year-old man sustains an injury to the anterior aspect of his right elbow while trying
to lift a heavy load 3 days ago. He has ecchymosis in the anterior and medial elbow regions and has difficulty
with resisted forearm supination with the elbow in a flexed position. A diagnosis of an acute distal biceps
tendon rupture is made and surgical treatment is chosen. The anatomic relationship of the distal biceps tendon
to the median nerve and recurrent radial artery within the antecubital fossa is such that the biceps tendon
travels
A. lateral (radial) to the median nerve and posterior (deep) to the recurrent radial artery.
B. lateral (radial) to the median nerve and anterior (superficial) to the recurrent radial artery.
C. medial (ulnar) to the median nerve and posterior (deep) to the recurrent radial artery.
D. medial (ulnar) to the median nerve and anterior (superficial) to the recurrent radial artery.

PREFERRED RESPONSE: A
DISCUSSION:
During surgical repair of a distal biceps tendon rupture, regardless of the surgical approach or technique, an
understanding of the regional anatomy is important. The tendon passes distally into the antecubital fossa. The
antecubital fossa is defined by the brachioradialis radially and the pronator teres ulnarly. A sheath surrounds
the biceps tendon as it passes through the antecubital fossa toward its insertion on the radial tuberosity. The
lateral antebrachial cutaneous nerve lies superficially in the subcutaneous tissue of the antecubital fossa. The
nerve parallels the brachioradialis. While still superficial, the tendon is contiguous with the lacertus fibrosus
that becomes confluent medially with the fascia overlying the flexor-pronator mass. The brachial artery lies
just beneath the lacertus fibrosus at the level of the elbow flexion crease. The tendon travels just lateral (radial)
to the median nerve within the antecubital fossa and passes posterior (deep) to the recurrent radial artery before
it attaches to the radial tuberosity. Full forearm supination allows visualization of the tendinous insertion on
the radial tuberosity.

Question 13 of 100
A 33-year old man sustains a posterior elbow dislocation after a fall. Attempts at closed reduction result in
recurrent instability. What is the most common ligamentous injury found at the time of surgical stabilization?
A. Midsubstance tear of the lateral ulnar collateral ligament
B. Proximal avulsion of the ulnar collateral ligament
C. Proximal avulsion of the lateral ulnar collateral ligament
D. Distal bony avulsion of the ulnar collateral ligament from the sublime tubercle

PREFERRED RESPONSE: C
DISCUSSION:
Classic posterior elbow dislocations result from a posterolateral rotatory mechanism, whereby the hand is
fixed (typically on the ground) while the weight of the body creates a valgus and external rotation moment on
the elbow. This results first in tearing of the lateral collateral ligament that proceeds medially through the
anterior and posterior joint capsules, ending with potential involvement of the ulnar collateral ligament (but
this is not universal). McKee and associates assessed the lateral soft-tissue injury pattern of elbow dislocations
with and without associated fractures at the time of surgery. Injury to the lateral collateral ligament complex
was seen in every case, with avulsion from the distal humerus as the most common finding. Midsubstance
tears, proximal avulsions, and distal bony avulsions of the ulnar collateral ligament are less common.

Question 14 of 100
Figures 1 and 2 are the radiographs of a 61-year-old woman with a left elbow injury after a fall onto her
outstretched hand. She denies any previous injury to her elbow. She undergoes a closed reduction of her elbow
in the emergency department. What is the most appropriate next step in definitive management?

A. Semiconstrained total elbow arthroplasty


B. Open reduction internal fixation (ORIF) of coronoid and radial head
C. ORIF of coronoid, radial head replacement, and lateral collateral ligament repair
D. ORIF of coronoid, radial head resection, and lateral collateral ligament repair
PREFERRED RESPONSE: C
DISCUSSION:

Question 15 of 100
A 35-year-old man falls off of a roof and sustains an extra-articular supracondylar elbow fracture. He had
normal sensation in all fingers after the injury and before undergoing surgery to repair the fracture. The ulnar
nerve was not transposed but was inspected prior to wound closure. Ten days after surgery, the patient has
numbness in his small finger and is unable to cross his fingers. His elbow range of motion is 40° to 100°. What
is the next appropriate step in management?
A. Elbow splint at 40° for 6 weeks
B. Electromyography (EMG)
C. Exploration of ulnar nerve and transposition
D. Continued observation

PREFERRED RESPONSE: D
DISCUSSION:
This patient has an early postsurgical ulnar nerve palsy. The causes of this injury are laceration of the nerve
during surgery, entrapment of the nerve in the fracture or hardware, or traction injury during surgery. If the
orthopaedic surgeon is sure that the nerve was not lacerated at the end of the case or entrapped in the hardware,
then the nerve is probably intact and will recover. Observation is the best treatment in this case because the
nerve was checked before wound closure. Elbow splinting has not been shown to help with postsurgical nerve
recovery. EMG findings may not be accurate this early following the injury.
Question 16 of 100
Figures 1 and 2 are the radiographs of a 64-year-old woman with a history of rheumatoid arthritis (RA) who
complains of right elbow pain. She has been treated with tumor necrosis factor-alpha inhibitors and oral
corticosteroids for several years. What process is primarily responsible for the radiographic joint destruction?

A. Traumatic insult resulting in complement activation


B. Mutation in the rheumatoid factor gene
C. Osteoblast paracrine signaling resulting in proteolytic collagen degradation
D. Inflammation resulting in a hyperplastic synovial joint lining
PREFERRED RESPONSE: D
DISCUSSION:
RA is a systemic inflammatory disorder marked by erosive arthritis in multiple joints. Elbow involvement is
common. The pathologic lesion in RA is pannus, a hyperplastic synovial proliferation that ultimately results
in proteoglycan and collagen digestion. Rheumatoid factor mutations, traumatic insults resulting in
complement activation, and osteoblast paracrine signaling are not involved in the pathologic process. The
Larsen classification assesses the progression of rheumatoid changes in the elbow. Stage I is characterized by
osteopenia without joint space narrowing. Stage II indicates joint space narrowing but a normal joint contour.
Stage III is marked by joint space loss. This patient has stage IV disease, as seen by the advanced erosive
changes with trochlear groove deepening and resulting deformity. Stage V is ankylosis.

Question 17 of 100
Figures 1 through 3 are the radiographs and MRI scan of a 13-year-old girl who has had right lateral elbow
pain and “popping” for 5 months. She has a history of competitive gymnastics for 5 years, which she stopped
participating in 2 years ago. She has since been pitching in softball. Her pain is getting worse, and she has not
played any sports for the last 2 months. She has undergone a course of physical therapy. On examination, she
lacks 5° of elbow extension. What is the next most appropriate step in management?
A.Application of dynamic hinged elbow brace
B.Arthroscopic debridement and capitellar microfracture
C.Lateral collateral ligament reconstruction
D. Debridement and release of extensor origin
PREFERRED RESPONSE: B
DISCUSSION:
The radiographs and MRI reveal a defect in the capitellum compatible with osteochondritis dissecans
(OCD). The initial treatment for an OCD lesion would be cessation of the offending activity with gradual
return to the activity over the course of 6 to12 weeks. In the setting of failed nonsurgical treatment,
surgical intervention may be considered. The patient has already attempted cessation of sporting
activities, as well as physical therapy; as such, a hinged elbow brace would not be of benefit. For the
smaller, unstable OCD lesions, arthroscopic debridement and microfracture is the most appropriate
treatment option. This would be followed by early protected range-of-motion exercises. The MRI does
not show evidence for a lateral collateral ligament injury or lateral epicondylitis. As a result, lateral
collateral ligament reconstruction or debridement of the extensor origin would not be indicated for this
patient.

Question 18 of 100
A 41-year-old right-hand-dominant man has been treated nonsurgically for right elbow arthritis. His
radiographs reveal end-stage ulnohumeral arthritis with complete loss of the joint space. He reports pain during
the mid-arc of elbow flexion and extension. During the last 8 years, he has attempted activity modification,
medication, physical therapy, and multiple cortisone injections. His symptoms have progressed, resulting in
constant pain, loss of a functional range of motion, and an inability to perform many activities of daily living.
Secondary to his age and activity demands, he undergoes a soft-tissue interposition arthroplasty of his elbow
with an Achilles allograft. Which presurgical finding correlates with elevated risk for postsurgical
complications?
A. Inflammatory elbow arthritis
B. A presurgical flexion-extension elbow arc of approximately 50°
C. Retained distal humerus hardware on presurgical radiographs
D. Evidence of presurgical elbow instability

PREFERRED RESPONSE: D
DISCUSSION:
End-stage posttraumatic or inflammatory elbow arthritis in active, high-demand patients remains difficult to
treat. Traditional total elbow arthroplasty is discouraged in this demographic secondary to concerns about
implant longevity. Soft-tissue interposition arthroplasty does not necessitate the same activity and weight
restrictions for patients after surgery and remains a reasonable salvage procedure. Larson and Morrey
published their findings on 38 patients with a mean age of 39 years following soft-tissue interposition
arthroplasty for posttraumatic and inflammatory end-stage elbow arthritis. These investigators reported a
significant improvement in Mayo Elbow Performance Score in addition to improvement in the flexion-
extension arc from 51° to 97° after surgery. They reported worse results and elevated incidence of
complications for patients with presurgical elbow instability upon examination; retained hardware from prior
surgery was not deemed a contraindication.

Question 19 of 100
Figures 1 and 2 are the radiographs of a 48-year-old right-hand dominant man who has had elbow pain and
stiffness for many years. The patient enjoys recreational softball and weightlifting, and the pain has gotten
severe enough that it has begun to interfere with his activities of daily living. On examination, he has a range
of motion from 40° to 110°, with pain with terminal extension and flexion, but no pain in the mid-range of
motion. He had one corticosteroid injection last year, which temporarily improved his pain but did not improve
his motion. What is the most appropriate treatment option for this patient?

A. Isolated radial head excision


B. Arthroscopic osteocapsular arthroplasty
C. Total elbow arthroplasty
D. Distal humerus hemiarthroplasty
PREFERRED RESPONSE: B
DISCUSSION:
This patient is relatively young, active, and has evidence of elbow osteoarthritis (OA). Large osteophytes in
the anterior and posterior compartments of his elbow are limiting motion. He has failed non-operative
treatment, and surgery is indicated. Arthroscopic osteocapsular arthroplasty allows removal of impinging
osteophytes and release of hypertrophied capsule and has been shown to be effective at relieving pain and
improving motion in patients with OA. Additionally, avoiding prosthetic replacement would allow the patient
to continue his active lifestyle. An open elbow release would be another excellent treatment option. Isolated
radial head excision would not be indicated, because it would increase contact forces across the ulnohumeral
joint and cause little improvement in symptoms. Total elbow arthroplasty is not a good option in a young,
active patient. Distal humerus hemiarthroplasty has been described for certain patients with unreconstructable
distal humerus fractures, but it is not a good option for treatment of OA.
Question 20 of 100
A 32-year-old man sustains elbow trauma in a motor vehicle collision and has medial elbow pain, swelling,
and bruising. A 3D-CT scan with representative images are shown in Figures 1 and 2.

What function does the ligamentous structure attached to the ulnar fracture fragment provide?

A. Tertiary restraint to posterior subluxation of the ulna at the ulnohumeral joint


B. Primary restraint to valgus stress at the elbow
C. Secondary restraint to posterior elbow subluxation
D. Secondary restraint to anterior subluxation of the radial head
PREFERRED RESPONSE: B
DISCUSSION:
The medial collateral ligament of the elbow inserts into the sublime tubercle, which is shown as a fracture
fragment on the 3D-CT scan. The medial collateral ligament is the primary restraint to valgus stress of the
elbow. Secondary restraints to elbow instability are the radiohumeral articulation, the common flexor-pronator
tendon, the common extensor tendon, and the elbow capsule. The primary restraint to posterolateral rotatory
instability is the lateral collateral ligament that originates on the lateral epicondyle and inserts on the crista
supinatoris of the ulna. There are no described "tertiary" restraints to the ulnohumeral joint. The anteromedial
coronoid facet is part of the coronoid, which extends more lateral and anterior than the anteromedial facet.
The anteromedial facet represents the critical weight-bearing portion of the ulnohumeral joint. Damage to this
structure causes posteromedial subluxation that often results in severe progressive arthritis. The coronoid is
the larger structure of which the anteromedial coronoid facet is a portion. The posteromedial coronoid facet
does not appear to be critical in weight bearing. The radial notch is not associated with increased stress with
weight bearing. The treatment of displaced fractures of this structure is ORIF utilizing buttress plating. Closed
treatment is acceptable only for nondisplaced fractures with appropriate radiographic follow-up. Suture
fixation is not advocated because of inadequate strength.

Question 21 of 100
A 23-year-old collegiate gymnast sustains a rupture of his medial collateral ligament of the elbow when he
falls off the parallel bars. On physical examination, he has instability to valgus stress and tenderness along the
medial elbow. Radiographs show no fracture. Which component of the medial collateral ligament of the elbow
is the dominant restraint to valgus stress?
A. Transverse ligament
B. Anterior band of the medial collateral ligament
C. Posterior band of the medial collateral ligament
D. Posterior capsule

PREFERRED RESPONSE: B
DISCUSSION:
The anterior bundle of the medial collateral ligament is the prime stabilizer against valgus stress. The posterior
bundle, which originates on the medial epicondyle and inserts broadly along the medial edge of the trochlea
from the sublime tubercle posteriorly, has stress only in elbow flexion. The transverse band, which originates
on the posteromedial olecranon and inserts on the sublime tubercle, deepens the trochlea, but neither the band
nor the posterior capsule provides significant restraint. The lateral collateral ligament, which originates from
the lateral epicondyle and inserts on the crista supinatoris of the ulna, is the prime stabilizer of varus stress
and posterolateral rotatory subluxation.

Question 22 of 100
A 13-year-old pitcher reports the immediate onset of medial elbow pain after throwing a pitch. Upon
examination, the patient is tender to palpation at the medial epicondyle and has pain and instability with valgus
testing of the elbow. If the patient were a college pitcher with a similar clinical presentation and physical
examination, what anatomic structure would most likely be injured?
A. Ulnar collateral ligament (UCL)
B. Pronator teres
C. Ligament of Struthers
D. Lateral collateral ligament

PREFERRED RESPONSE: A
DISCUSSION:
The patient has an acute avulsion fracture of the medial epicondyle, which can occur in response to the valgus
load placed on the elbow while throwing. Diagnosis is confirmed by radiograph, with comparison views of
the uninjured elbow to evaluate for physeal closure versus injury. In older pitchers, the UCL fails rather than
the bone of the medial epicondyle. Advanced imaging may be necessary to confirm the diagnosis of an UCL
injury and/or bony injury.
Question 23 of 100
A 55-year-old woman develops posttraumatic arthritis in the elbow following a distal humerus fracture. What
is the most likely mid-term (5-10 years after surgery) complication following semiconstrained total elbow
arthroplasty (TEA)?
A. Bushing wear
B. Infection
C. Aseptic component loosening
D. Component fracture

PREFERRED RESPONSE: A
DISCUSSION:
TEA has been described for posttraumatic arthritis of the elbow and typically involves a young patient
population with multiple previous operations on the affected elbow. Morrey and Schneeberger found aseptic
component loosening to be uncommon (<10% of patients) and usually occurring >10 years after surgery.
Prosthetic fracture, usually of the ulnar component, is also a late-term finding. Infection is the most common
mode of early failure but usually occurs within the first 5 years and has an overall rate of approximately 5%.
Bushing wear has been reported as the most common cause of mechanical TEA failure in this population at
intermediate-term follow-up.

Question 24 of 100
A 13-year-old pitcher reports the immediate onset of medial elbow pain after throwing a pitch. Upon
examination, the patient is tender to palpation at the medial epicondyle and has pain and instability with valgus
testing of the elbow. The diagnostic modality selected in the prior question would be most useful to evaluate
for
A. apophyseal injury.
B. osteochondral defect.
C. presence of hematoma.
D. valgus overload injury.

PREFERRED RESPONSE: A
DISCUSSION:
The patient has an acute avulsion fracture of the medial epicondyle, which can occur in response to the valgus
load placed on the elbow while throwing. Diagnosis is confirmed by radiograph, with comparison views of
the uninjured elbow to evaluate for physeal closure versus injury. In older pitchers, the UCL fails rather than
the bone of the medial epicondyle. Advanced imaging may be necessary to confirm the diagnosis of an UCL
injury and/or bony injury.

Question 25 of 100
A 26-year-old mixed martial arts fighter sustains a posterolateral elbow dislocation. The primary stabilizers
of the elbow joint are the
A. radiocapitellar joint, the posterior band of the medial collateral ligament, and the annular ligament.
B. ulnohumeral joint, the anterior band of the medial collateral ligament, and the lateral ulnar collateral
ligament.
C. radiocapitellar joint, the anterior band of the medial collateral ligament, and the radial collateral
ligament.
D. ulnohumeral joint, the anterior band of the medial collateral ligament, and the posterior band of the
medial collateral ligament.
PREFERRED RESPONSE: B
DISCUSSION:
The primary stabilizers of the elbow are the ulnohumeral joint, the lateral collateral ligament (lateral
epicondyle to the crista supinatoris), and the anterior band of the medial collateral ligament (anterior inferior
medial epicondyle to the sublime tubercle). Secondary stabilizers are the radial head, the common flexor and
extensor origins, and the joint capsule. The muscles that cross the elbow joint act as dynamic stabilizers.

Question 26 of 100
A 51-year-old butcher has an 18-month history of recalcitrant medial elbow pain, which is affecting his
occupational demands. He describes the pain as mainly anterior and distal to the medial epicondyle. His
symptoms are exacerbated with resisted wrist flexion and forearm pronation. On examination, he is also found
to have a positive Tinel’s sign at the elbow with weakness of intrinsic strength. He has attempted physical
therapy, activity modification, bracing, and anti-inflammatory medication without any significant
improvement. Presurgical counseling should include the understanding that
A. concomitant ulnar neuropathy is a potential poor prognostic factor.
B. a change in occupation will likely be required after surgery.
C. weakness in wrist flexion strength will result postoperatively.
D. prior corticosteriod injections are a potential poor prognostic factor.

PREFERRED RESPONSE: A
DISCUSSION:
Although less common in comparison with lateral elbow tendinopathy, medial elbow tendinopathy remains a
significant cause of elbow disability. Fortunately, most patients can anticipate resolution of symptoms with
nonsurgical management. For patients with recalcitrant symptoms, surgical intervention should be discussed
as a treatment alternative. The literature reports successful results with surgical intervention via debridement
of pathologic tissue, release of the flexor carpi radialis - pronator teres origin, and/or repair of the flexor carpi
radialis - pronator teres origin. Several authors have raised concern of the impact of concomitant ulnar
neuropathy on results following surgical treatment for medial epicondylitis. Kurvers and Verhaar and Gabel
and Morrey, among others, have reported a statistically significant association between concomitant ulnar
neuropathy and worse outcomes following surgery. Most patients can anticipate a return to prior activity levels
after surgery without any consistently reported loss of flexor/pronator strength. Prior corticosteroid injections
have not been found to impact results.

Question 27 of 100
A 17-year-old girl develops chronic posterolateral rotatory instability (PLRI) of the elbow following closed
treatment of an elbow dislocation. Advanced imaging reveals incompetence of the lateral collateral ligament
complex, and ligament reconstruction is planned. Examination under anesthesia is performed with the forearm
in maximal supination and valgus force applied to the elbow, demonstrated in Video 1(The pivot shift test)
As the elbow is brought through a range of motion assessment, the radial head is

A. dislocating posteriorly in extension and reducing in flexion.


B. dislocating posteriorly in flexion and reducing in extension.
C. dislocating anteriorly in extension and reducing in flexion.
D. dislocating anteriorly in flexion and reducing in extension.
PREFERRED RESPONSE: A
DISCUSSION:
PLRI of the elbow is the most common form of chronic elbow instability. The mechanism occurs following a
fall onto an outstretched hand, where a valgus force is applied to the elbow and the forearm rotates into
progressive supination. This allows the radial head to translate posterior to the capitellum, with progressive
injury from lateral to medial sides of the elbow. The pivot shift test is a useful examination maneuver to
confirm the presence of PLRI. With the forearm in maximal supination and valgus stress applied to the elbow,
the radial head is forced posterior to the capitellum as the elbow is brought into progressive extension,
revealing a dimple on the lateral aspect of the elbow. This typically occurs at roughly 30⁰ of flexion. As the
elbow is flexed, the radial head reduces.

Question 28 of 100
A 13-year-old pitcher reports the immediate onset of medial elbow pain after throwing a pitch. Upon
examination, the patient is tender to palpation at the medial epicondyle and has pain and instability with valgus
testing of the elbow. What would be the most appropriate initial diagnostic test for this patient?
A. MRI arthrogram
B. CT scan with 3-dimensional reconstructions
C. Plain radiographs of both elbows
D. Ultrasonography

PREFERRED RESPONSE: C
DISCUSSION:
The patient has an acute avulsion fracture of the medial epicondyle, which can occur in response to the valgus
load placed on the elbow while throwing. Diagnosis is confirmed by radiograph, with comparison views of
the uninjured elbow to evaluate for physeal closure versus injury. In older pitchers, the UCL fails rather than
the bone of the medial epicondyle. Advanced imaging may be necessary to confirm the diagnosis of an UCL
injury and/or bony injury.
Question 29 of 100
A 45-year-old construction worker sees a surgeon 23 days after sustaining an eccentric injury to his dominant
right elbow. An MRI demonstrates a distal biceps tendon rupture with 5 cm of proximal retraction. In the
operating room, the surgeon encounters good tissue quality but finds that primary repair can only be performed
with the elbow hyperflexed to 70°. What is the best next step?
A. Proceed with primary repair with the elbow hyperflexed
B. Use interposition allograft to reconstruct with elbow in extension
C. Tenodese distal biceps tendon to underlying brachialis muscle
D. Forego primary repair, but perform stump debridement

PREFERRED RESPONSE: A
DISCUSSION:
Distal biceps ruptures, although relatively less common in comparison with other upper extremity tendon
injuries, still garner considerable attention in the orthopaedic literature. The mechanism of injury typically
results from an eccentric extension load to a flexed elbow. A biceps-deficient arm can result in up to 40% loss
of supination strength and up to 80% loss of supination endurance. A delay in diagnosis can compromise the
ability to reduce the tendon back to its anatomic insertion without having to hyperflex the elbow. Current
literature confirms the ability to safely proceed with primary repair even with the elbow flexed up to 100°
without fear of developing a flexion contracture. With time, patients can anticipate restoration of full elbow
extension. An interposition graft should be used for a poor residual tendon quality stump <4 cm in length and
in cases of delay to surgery of >6 weeks. Biceps to brachialis tendon transfer does not restore supination
strength. Isolated debridement of the distal tendon would not be an appropriate treatment.

Question 30 of 100
A 45-year-old construction worker has right elbow pain and swelling following a fall from a step ladder onto
his outstretched hand. Figures 1 and 2 are his radiograph and 3D-CT scan. Examination under anesthesia
reveals widening of the radiocapitellar joint space with varus stress. Surgical intervention is undertaken for
open reduction internal fixation (ORIF) of the coronoid fragment. Following fixation, the elbow remains
unstable with persistent varus laxity. What is the best next step?

A. Application of a long-arm splint with 4 to 6 weeks of immobilization


B. Application of a dynamic external fixator
C. Transarticular pinning of the ulnohumeral joint for 4 to 6 weeks
D. Open repair of the lateral collateral ligament complex
PREFERRED RESPONSE: D
DISCUSSION:
Posteromedial rotatory instability of the elbow is typically associated with a fall backwards onto an
outstretched arm, resulting in varus stress across the elbow and progressive pronation of the forearm. As the
coronoid translates posterior to the trochlea, the anteromedial facet is often fractured. The lateral ulnar
collateral ligament is frequently compromised as a result of the varus force. The degree of elbow instability
relates to both the size of the coronoid fragment and the associated soft-tissue injuries about the elbow. The
images reveal a displaced O’Driscoll type 2 coronoid fracture that is amenable to ORIF. If this fails to restore
stability to the elbow, the next most appropriate intervention is to repair the lateral collateral ligament complex,
an injury that is suggested by the widened radiocapitellar joint space on preoperative evaluation. A dynamic
external fixator can be used in the setting of grossly unstable elbows, but all correctible instability lesions
should be addressed before determining if this is necessary. Immobilization or temporary transarticular
pinning would be unlikely to adequately restore stability to the elbow.

Question 31 of 100
Figure 1 is the MRI scan of a 25-year-old left-hand dominant minor league pitcher who has elbow pain during
pitching that has gotten worse for the past several months. He fails nonoperative treatment and undergoes
surgery to address the problem. What is the most common complication of this procedure?

A. Ulnar nerve neuropraxia


B. Flexor pronator mass avulsion
C. Posterolateral rotatory instability
D. Symptomatic hardware
PREFERRED RESPONSE: A
DISCUSSION
The MRI scan shows evidence of a medial collateral ligament (MCL) injury. In a patient with a chronic MCL
injury that has failed non-operative treatment, MCL reconstruction would be indicated. Initial MCL
reconstruction technique involved routine transposition of the ulnar nerve and detachment of the flexor-
pronator mass. Subsequent modification of surgical techniques has been made to attempt to minimize
complications by avoiding routine ulnar nerve transposition and performing a muscle-splitting approach.
Regardless, transient ulnar nerve neuropraxia remains the most common complication, and patients should be
counseled about its occurrence. Flexor pronator mass avulsion is more likely with a muscle-detaching
approach but is not more common than ulnar nerve neuropraxia. Posterolateral rotatory instability is a
complication of lateral collateral ligament repair or reconstruction, not medial collateral ligament
reconstruction. Symptomatic hardware is not a common complication.

Question 32 of 100
A 32-year-old man sustains elbow trauma in a motor vehicle collision and has medial elbow pain, swelling,
and bruising. A 3D-CT scan with representative images are shown in Figures 1 and 2.Definitive treatment of
this fracture should consist of

A. closed reduction, limited immobilization (1-2 weeks), and early functional rehabilitation.
B. limited immobilization in a long-arm cast (4 weeks) and early functional rehabilitation.
C. open reduction and internal fixation (ORIF).
D. open reduction, capsular repair, and suture fixation of the bony fragment and ligament.
PREFERRED RESPONSE: C
DISCUSSION
The medial collateral ligament of the elbow inserts into the sublime tubercle, which is shown as a fracture
fragment on the 3D-CT scan. The medial collateral ligament is the primary restraint to valgus stress of the
elbow. Secondary restraints to elbow instability are the radiohumeral articulation, the common flexor-pronator
tendon, the common extensor tendon, and the elbow capsule. The primary restraint to posterolateral rotatory
instability is the lateral collateral ligament that originates on the lateral epicondyle and inserts on the crista
supinatoris of the ulna. There are no described "tertiary" restraints to the ulnohumeral joint. The anteromedial
coronoid facet is part of the coronoid, which extends more lateral and anterior than the anteromedial facet.
The anteromedial facet represents the critical weight-bearing portion of the ulnohumeral joint. Damage to this
structure causes posteromedial subluxation that often results in severe progressive arthritis. The coronoid is
the larger structure of which the anteromedial coronoid facet is a portion. The posteromedial coronoid facet
does not appear to be critical in weight bearing. The radial notch is not associated with increased stress with
weight bearing. The treatment of displaced fractures of this structure is ORIF utilizing buttress plating. Closed
treatment is acceptable only for nondisplaced fractures with appropriate radiographic follow-up. Suture
fixation is not advocated because of inadequate strength.

Question 33 of 100
A 53-year-old man complains of recurrent lateral elbow pain. He was surgically treated approximately one
year ago with some improvement in his direct lateral elbow pain. He now reports new-onset discomfort at the
posterolateral elbow, as well as difficulty when pushing himself up from a chair. On examination, he has a
well-healed 6-cm incision over the lateral epicondyle with full active and passive range of motion. He has
pain with palpation along the posterior lateral elbow and a positive posterior drawer test. Radiographs are
unremarkable. What is the best next step?
A. Platelet-rich plasma
B. Physical therapy
C. Lateral epicondyle debridement
D. Lateral collateral ligament reconstruction

PREFERRED RESPONSE: D
DISCUSSION
Lateral elbow tendinopathy remains a frequently encountered pathology of the elbow. Open or arthroscopic
lateral epicondyle debridement can be considered for patients with refractory symptoms. With either
technique, the lateral collateral ligament complex of the elbow is at risk for compromise, with excessive
debridement distal and posterior to the center of rotation of the capitellum. When injured, patients often
complain of pain around the posterior lateral elbow, which is commonly misdiagnosed as recurrent lateral
epicondylitis. The push-up test (apprehension using the supinated forearm to push up from a chair) is a typical
examination finding, along with a positive posterior drawer test. Patients may also develop posterior lateral
instability of the elbow, for which the recommended treatment is lateral collateral ligament reconstruction.

Question 34 of 100
A 32-year-old man sustains elbow trauma in a motor vehicle collision and has medial elbow pain, swelling,
and bruising. A 3D-CT scan with representative images are shown in Figures 1 and 2.The critical weight-
bearing portion of the elbow joint that is damaged in this fracture is the

A. anteromedial coronoid facet.


B. Posteromedial olecranon facet.
C. coronoid.
D. radial notch.
PREFERRED RESPONSE: A
DISCUSSION
The medial collateral ligament of the elbow inserts into the sublime tubercle, which is shown as a fracture
fragment on the 3D-CT scan. The medial collateral ligament is the primary restraint to valgus stress of the
elbow. Secondary restraints to elbow instability are the radiohumeral articulation, the common flexor-pronator
tendon, the common extensor tendon, and the elbow capsule. The primary restraint to posterolateral rotatory
instability is the lateral collateral ligament that originates on the lateral epicondyle and inserts on the crista
supinatoris of the ulna. There are no described "tertiary" restraints to the ulnohumeral joint. The anteromedial
coronoid facet is part of the coronoid, which extends more lateral and anterior than the anteromedial facet.
The anteromedial facet represents the critical weight-bearing portion of the ulnohumeral joint. Damage to this
structure causes posteromedial subluxation that often results in severe progressive arthritis. The coronoid is
the larger structure of which the anteromedial coronoid facet is a portion. The posteromedial coronoid facet
does not appear to be critical in weight bearing. The radial notch is not associated with increased stress with
weight bearing. The treatment of displaced fractures of this structure is ORIF utilizing buttress plating. Closed
treatment is acceptable only for nondisplaced fractures with appropriate radiographic follow-up. Suture
fixation is not advocated because of inadequate strength.

Question 35 of 100
A 54-year-old woman undergoes an interposition arthroplasty that fails and requires conversion to a total
elbow arthroplasty. She has progressive elbow pain and radiographic loosening. Erythrocyte sedimentation
rate and C-reactive protein are normal. Joint aspiration is positive for Staphylococcus epidermidis. What
surgical treatment would best optimize function and decrease risk of recurrence?
A. Resection arthroplasty
B. Single-stage revision total elbow arthroplasty
C. Two-stage revision elbow arthroplasty
D. Aggressive arthroscopic debridement and retention of components

PREFERRED RESPONSE: C
DISCUSSION
The most reliable surgical option in this case for eradicating a deep infection following a total elbow
arthroplasty is a two-stage revision. One study, however, reported that staged reimplantation of an infected
total elbow replacement could be successful in the setting of organisms other than S epidermidis. Arthroscopic
debridement is not a viable option with poorly fixed or loose components. A single-stage revision, while
considered an option in hip and knee arthroplasty, has not been definitively proven to be an option for revision
total elbow arthroplasty. Single-stage revision has shown moderate success in the setting of Staphylococcus
aureus infections, although with only short-term follow-up. A resection arthroplasty would likely be
successful in managing the deep infection but would not optimize the functional result. Resection arthroplasty
is best reserved for low-demand or infirm patients.

Question 36 of 100
Figures 1 and 2 are the radiographs of a 47-year-old right-hand-dominant active man with a 10-year history
of progressive right elbow pain associated with stiffness. He previously underwent collateral ligament
reconstruction. He has pain throughout his range-of-motion arc, which currently measures 20° of extension to
80° of flexion. Initial treatment with nonsteroidal anti-inflammatory medication, physical therapy, cortisone
injections, and arthroscopic debridement has failed to provide relief of his symptoms and improvement in
function. What is the most appropriate treatment if instability is present at the time of evaluation?

A. TEA
B. Distal humeral replacement arthroplasty
C. Arthroscopic release with debridement
D. Soft-tissue interposition arthroplasty
PREFERRED RESPONSE: A
DISCUSSION
The radiographs reveal ulnohumeral arthrosis with relative sparing of the radiocapitellar articulation
secondary to underlying osteoarthritis. Arthrosis of the elbow joint in this young and active patient presents a
treatment dilemma for the surgeon. Interposition arthroplasty allows for improved function with pain relief
and no weight-lifting restrictions, as required with TEA. This option is an intermediate procedure that
preserves bone stock and allows for conversion to a TEA if necessary. Conventional TEA would provide pain
relief with improved range of motion, but activity limitation and lifetime weight restrictions make this an
undesirable option. Arthroscopic debridement is not an option, considering the previous failure from this
modality. Contraindications for soft-tissue interposition arthroplasty include elbow instability, active
infection, and pain without motion loss. Common complications associated with this procedure include
instability, infection, ulnar neuropathy, bone resorption, and heterotopic bone formation.

Question 37 of 100
Figures 1 and 2 are the radiographs of a 47-year-old right-hand-dominant active man with a 10-year history
of progressive right elbow pain associated with stiffness. He previously underwent collateral ligament
reconstruction. He has pain throughout his range-of-motion arc, which currently measures 20° of extension to
80° of flexion. Initial treatment with nonsteroidal anti-inflammatory medication, physical therapy, cortisone
injections, and arthroscopic debridement has failed to provide relief of his symptoms and improvement in
function. What is the most appropriate next treatment step for this patient?

A. Total elbow arthroplasty (TEA)


B. Distal humeral replacement arthroplasty
C. Arthroscopic release with debridement
D. Soft-tissue interposition arthroplasty
PREFERRED RESPONSE: D
DISCUSSION
The radiographs reveal ulnohumeral arthrosis with relative sparing of the radiocapitellar articulation
secondary to underlying osteoarthritis. Arthrosis of the elbow joint in this young and active patient presents a
treatment dilemma for the surgeon. Interposition arthroplasty allows for improved function with pain relief
and no weight-lifting restrictions, as required with TEA. This option is an intermediate procedure that
preserves bone stock and allows for conversion to a TEA if necessary. Conventional TEA would provide pain
relief with improved range of motion, but activity limitation and lifetime weight restrictions make this an
undesirable option. Arthroscopic debridement is not an option, considering the previous failure from this
modality. Contraindications for soft-tissue interposition arthroplasty include elbow instability, active
infection, and pain without motion loss. Common complications associated with this procedure include
instability, infection, ulnar neuropathy, bone resorption, and heterotopic bone formation.

Question 38 of 100
Figure 1 is the MRI arthrogram of a 21-year-old professional baseball pitcher who complains of right elbow
pain after pitching a game 3 months ago. He had initially been treated with rest and forearm strengthening. He
now complains of persistent pain along his medial elbow during the long toss portion of his throwing program.
What is the most appropriate treatment at this time?

A. Arthroscopic debridement and capsular release


B. Common flexor pronator re-attachment
C. Medial ulnar collateral ligament (MUCL) reconstruction
D. Extensor carpi radialis brevis release
PREFERRED RESPONSE: C
DISCUSSION
MUCL reconstruction, using either ipsilateral palmaris longus, hamstring autograft, or allograft tendon is
indicated for ulnar collateral ligament (UCL) injuries that fail nonsurgical management. The MRI arthrogram
shows a rupture of the UCL from its ulnar insertion, with the classic T sign of contrast extravasation. The MRI
study does not show any ulnohumeral or radiocapitellar osteoarthritis to suggest radial head resection or
debridement/capsular release as indicated procedures; the patient’s symptoms are medial, precluding the need
for a release of the extensor carpi radialis brevis. Finally, the common origin of the flexor pronator mass
appears intact on the given arthrogram image.

Question 39 of 100
A 35-year-old construction worker falls from a ladder, sustaining an injury to his elbow. An MRI demonstrates
a rupture of his distal biceps tendon. He underwent a repair through a single transverse incision in the
antecubital fossa using the fixation as seen in Figures 1 and 2. Postoperatively, the patient reports mildly
decreased sensation over the anterolateral aspect of the forearm. Which other complication is this patient most
at risk for in the early postoperative period?
A. Proximal radioulnar synostosis
B. Heterotopic ossification
C. Radial neck fracture
D. Weak thumb and index interphalangeal joint flexion
PREFERRED RESPONSE: C
DISCUSSION
Injury to the lateral antebrachial cutaneous (LABC) nerve is the most common complication following distal
biceps tendon repair, primarily due to traction. The patient demonstrates some decreased sensation over the
anterolateral forearm, consistent with an LABC neurapraxia. Additionally, the figures show fixation with the
endobutton proximal to the radial tuberosity, in the region of the radial neck. This is associated with an
increased risk of iatrogenic fracture of the radial neck, which occurs during the drilling of the bone tunnel.
Heterotopic ossification and proximal radioulnar synostosis are more commonly associated with the dual-
incision approach. The anterior interosseous nerve is not an at-risk structure during distal biceps repair. The
posterior interosseous nerve (PIN) can be placed at risk during this procedure. The distance from the guide
pin and the nerve is decreased with more distal drilling and more radially directed drilling. In this case, the
PIN has not crossed posterior to the radius yet, making it less likely to be injured during placement of the
tunnel.

Question 40 of 100
A 20-year-old collegiate pitcher sustains a medial collateral ligament (MCL) rupture of his throwing elbow
for which surgical reconstruction is necessary. The goal of surgery is anatomic restoration of the MCL. Which
statement best describes the kinematics of the native MCL?
A. The posterior bundle demonstrates the greatest change in tension from flexion to extension.
B. The posterior bundle is isometric.
C. The anterior bundle becomes tight in flexion and lax in extension.
D. The anterior and posterior bundles are isometric.

PREFERRED RESPONSE: A
DISCUSSION
The anterior bundle is the most important portion of the complex when treating valgus instability of the elbow.
The ligament originates from the anteroinferior surface of the medial epicondyle. The anterior bundle inserts
on the medial border of the coronoid at the sublime tubercle. The anterior bundle of the medial collateral
ligament (MCL) is the primary restraint to valgus stress, and the radial head is a secondary restraint. With
anterior bundle sectioning, the resultant instability is most substantial between 60° and 70° and is lowest at
full extension and full flexion. True lateral radiographs reveal that the flexion-extension axis, or center of
rotation, of the elbow lies in the center of the trochlea and capitellum. The origin of the anterior bundle of the
MCL lies slightly posterior to the rotational center of the elbow. The anterior bundle is further divided into an
anterior band and a posterior band. The eccentric origin of these anterior bundle components in relation to the
rotational center through the trochlea creates a CAM effect during flexion and extension. The anterior band
tightens during extension, and the posterior band tightens during flexion. This reciprocal tightening of the two
functional components of the anterior bundle allows the ligament to remain taut throughout the full range of
flexion. Cadaver dissection studies have identified the origin and insertion of both the medial and lateral
stabilizing elbow ligaments. The anterior bundle of the MCL is isometric throughout the flexion/extension arc
of motion, making Response C incorrect. The posterior bundle of the MCL elongates with elbow flexion, so
Responses B and D are incorrect. The posterior bundle of the MCL also demonstrates the most change in
length from extension to flexion of all the elbow ligaments.

Question 41 of 100
Figure 1 is the MRI of a 45-year-old woman with a medical history significant for rheumatoid arthritis who
returns to your office with persistent right elbow pain. Her rheumatologist has maximized her disease-
modifying anti-rheumatoid drug regimen. She complains of diffuse joint pain and swelling. On examination,
she has a pronounced joint effusion, elbow flexion arc of 45°, and crepitus with forearm rotation. Her elbow
radiograph reveals preservation of her joint space. What is the most appropriate surgical treatment at this time?

A. Total elbow arthroplasty


B. Synovectomy with radial head resection
C. ynovectomy without radial head resection
D. Isolated radial head resection
PREFERRED RESPONSE: C
DISCUSSION
Rheumatoid arthritis remains a common inflammatory arthropathy that can lead to progressive synovitis of
the elbow joint. Patients often present with recalcitrant elbow pain and loss of motion. In the early stages, the
joint space can be fairly well preserved. With progressive synovitis, cartilage destruction leads to symmetric
joint space narrowing and joint destruction. For the younger patient with recalcitrant synovitis and a relatively
well-preserved joint space, open or arthroscopic synovectomy provides successful improvement for 70% to
80% of patients. In most cases, radial head resection is not required. Synovitis that encircles the radial head
and neck can lead to pain and crepitus with forearm rotation. Preserving the radial head prevents the rapid
progression of wear at the lateral ulnohumeral joint. A total elbow replacement, while a successful treatment
modality for the older, lower demand patient with rheumatoid arthritis, would not be appropriate for the
younger patient given the significant postoperative restrictions imparted.

Question 42 of 100
A 24-year-old right-hand-dominant professional baseball pitcher has valgus extension overload (VEO)
syndrome of the right elbow, as seen in Figure 1. Which letter in the Figure 1 corresponds to the typical area
of osteophyte formation in this condition?

A. A
B. B
C. C
D. D
PREFERRED RESPONSE: B
DISCUSSION
VEO is most commonly seen in throwers for whom valgus stress across the elbow causes impingement of the
posteromedial olecranon tip against the medial wall of the olecranon fossa. With repeated impingement, a
bony osteophyte may grow on the olecranon at the site of impingement in this posteromedial region of the
olecranon. Bony growth within the olecranon fossa also has been seen. The distinction between this condition
and ulnar collateral ligament (UCL) injury is difficult to make, but VEO often can be distinguished from UCL
injury by determining the exact location of pain a patient experiences. With VEO, the pain typically occurs
with direct palpation of the posterior medial tip of the olecranon. The valgus extension overload provocative
test also aids in diagnosis. A supervised physical therapy program and arthroscopic surgical decompression
when nonsurgical treatment is unsuccessful are typical treatments for this condition. Locations C and D
represent the origin and insertion, respectively, of the elbow medial collateral ligament (MCL), and, although
associated MCL pathology can exist in the setting of VEO syndrome, osteophyte formation is not typical in
these areas. Location A is the radial head, and although the radiocapitellar joint is a known secondary stabilizer
of elbow valgus stress, osteophyte formation in this area is less likely in this clinical scenario.

Question 43 of 100
Figures 1 and 2 show the radiograph and axial CT scan of a 56-year-old right-hand-dominant man who sustains
a right shoulder injury following a fall from a roof. He is seen in the emergency department and placed into a
sling. He denies any previous injury to the shoulder. His medical history is significant only for hypertension.
His arm is neurovascularly intact, and his deltoid is functioning. What is the most appropriate surgical option
at this point?

A. Open reduction and internal fixation (ORIF)


B. Anatomic total shoulder arthroplasty
C. Hemiarthroplasty
D. Reverse shoulder arthroplasty
PREFERRED RESPONSE: C
DISCUSSION
The radiograph and axial CT image demonstrate a displaced right proximal humerus fracture, which has a
head-splitting component. The fracture line extends through the greater and lesser tuberosities as well. The
head-splitting nature of the fracture increases the risk for the developement of avascular necrosis and,
potentially, failure of fixation. In the younger patient, <55 years, with a simple head-splitting fracture
(tuberosities intact), one might consider ORIF. However, in this case of an older patient, >55 years, with a
more complex head-splitting fracture pattern, a well-performed hemiarthroplasty provides a reliable clinical
outcome. Clinical outcomes following hemiarthroplasty rely on successful healing of the greater tuberosity.
In this case, the patient has no major medical comorbidities, has a non-comminuted greater tuberosity, and
sought treatment shortly after injury. These factors have been associated with improved tuberosity healing. In
an older patient with multiple medical comorbidities including osteopenia/osteoporosis, lower demand or with
a lengthy delay from injury to surgery, a reverse shoulder arthroplasty might be a reasonable option. Reverse
total shoulder arthroplasty has been shown to offer predictable functional results in the setting of fractures,
although with a higher complication rate.

Question 44 of 100
Figures 1 and 2 are the radiograph and MRI scan of a 40-year-old man who falls down a flight of stairs. His
upper arm is bruised and painful, and global weakness in the shoulder girdle function is noted. A radiograph
is ordered to rule out a fracture or dislocation. You should recommend
A. immediate open reduction and internal fixation of the fracture.
B. closed treatment with serial radiographs.
C. fracture fragment excision and deltoid repair.
D. rest, anti-inflammatory medications, and a home exercise program.
PREFERRED RESPONSE: D
DISCUSSION
The patient has an os acromiale. The type shown is of the meso-acromion. This is not an acute fracture; well
corticated ends are seen on the axillary radiograph, and there is no bone edema on the T2 axial MRI image. A
trial of nonsurgical care that includes rest, ice, and anti-inflammatory medications is recommended. If a patient
continues to have symptoms, an arthroscopic evaluation is needed to determine if the os acromiale is mobile
and if surgical fixation is appropriate.

Question 45 of 100
Surgical management of the fracture shown in Figure 1 will have what outcome compared with nonsurgical
management in a sling?

A. Increased rate of radiographic union


B. Increased amount of shortening
C. Decreased risk of infection
D. Improved shoulder forward elevation
PREFERRED RESPONSE: D
DISCUSSION
Multiple prospective randomized clinical trials have compared operative treatment of displaced midshaft
clavicle fractures with nonoperative management. While functional outcomes have not consistently been
improved with surgery, rates of radiographic union have consistently been shown to improve. Fracture
characteristics associated with an increased risk for nonunion of a midshaft clavicle fracture include
comminution of the fracture and significant shortening/displacement of the fracture (>1.5-2.0 cm). Patient
factors predicting nonunion include a smoking history.

Question 46 of 100
The fracture seen in Figure 1 is most likely associated with injury to what ligamentous structure?

A. Inferior glenohumeral ligament


B. Acromioclavicular (AC) ligaments
C. Coracoclavicular ligaments
D. Coracoacromial ligament
PREFERRED RESPONSE: C
DISCUSSION
The radiograph shows an extra-articular distal clavicle fracture lateral to the clavicular attachment point of the
coracoclavicular ligaments (conoid and trapezoid). However, unlike a scenario featuring a typical Neer type I
fracture, the interval between coracoid and clavicle is clearly widened, and there is marked fracture
displacement. It is clear that the coracoclavicular ligaments must also be torn. The inferior glenohumeral
ligament is important to glenohumeral joint stability but has no effect on the relationship between clavicle and
scapula. The AC ligaments are thickenings of the AC joint capsule. They have been shown to be responsible
for 90% of anteroposterior stability of the AC joint. The coracoclavicular ligaments are responsible for 77%
of stability for superior translation (as in this case). The coracoacromial ligament connects two parts of the
scapula (coracoid and acromion) and is part of the arch that supports the rotator cuff.

Question 47 of 100
A 55-year-old man falls from a ladder and dislocates his nondominant shoulder. He undergoes an
uncomplicated closed reduction under sedation in the emergency department. Postreduction radiographs
reveal a small Hill-Sachs lesion and no other bony abnormalities. Six weeks after the dislocation, the patient
has persistent pain at rest and forward elevation and external rotation weakness, but the remaining motor
function in the extremity and sensation are intact. What is the best next step?
A. Physical therapy with electrical stimulation and iontophoresis
B. Corticosteroid injection
C. MRI of the shoulder
D. Electromyography (EMG) of the arm

PREFERRED RESPONSE: C
DISCUSSION
For a patient >40 years of age who has persistent pain and weakness isolated to the rotator cuff following an
acute anterior shoulder dislocation, an MRI is indicated to evaluate rotator cuff integrity. EMG is not indicated
in this case because this patient has no evidence of distal motor functional abnormality and their sensation is
intact, thereby making a brachial plexus injury unlikely. Corticosteroid injections and physical therapy
modalities do not adequately address the concern over his potential for having sustained a rotator cuff tear.

Question 48 of 100
Figure 1 is the radiograph of a 27-year-old bicyclist who was involved in a crash. He has an isolated and closed
injury. He is neurovascularly intact in the upper extremity. The lateral fragment is displaced inferiorly by

A. gravity.
B. the trapezius.
C. the biceps.
D. the pectoralis minor.
PREFERRED RESPONSE: A
DISCUSSION
Open reduction and internal fixation with a plate and screw construct have been demonstrated to reduce
nonunion rate and improve outcomes compared with sling immobilization for displaced clavicle fractures.
Neurovascular injury and infection risk increase, however, with surgery. In the upright position, the weight of
the extremity inferiorly displaces the lateral segment.

Question 49 of 100
Figure 1 is the radiograph of a right-hand-dominant 70-year-old woman who arrives at the emergency
department with acute left shoulder pain following a fall down a flight of stairs. She expresses acute diffuse
left shoulder pain and swelling. Prior to her injury, she had fully active, painless shoulder range of motion.

Which radiographic parameter places this patient at the highest risk for osteonecrosis?
A. 3-part displaced valgus-type fracture pattern
B. 3-part displaced varus-type fracture pattern
C. Anterior fracture extension disrupting the bicipital groove
D. Posteromedial metaphyseal head extension of <8 mm
PREFERRED RESPONSE: D
DISCUSSION
Fractures of the proximal humerus are now the third most common fracture in patients >60 years of age. This
patient sustained a displaced, commonly described 3-part/4-part proximal humerus fracture. The number of
fracture fragments and angulation, as initially described by Codman and then Neer, does not necessarily help
to predict risk for subsequent AVN. Although the main blood supply to the humeral head historically was
believed to be a branch from the anterior circumflex, adequate perfusion can remain through the posteromedial
calcar following trauma. Hertel and associates reported that the most accurate predictor of ischemia was
whether the length of the metaphyseal head extension for the calcar segment was <8 mm. Locking plates have
provided surgical alternatives to many unstable fracture patterns previously considered ominous. Although
much enthusiasm remains for this implant choice, a relatively high level of complications has been reported
with their use. A multicenter study reported a 14% incidence of intra-articular screw perforation as the most
common complication. When patients are deemed poor candidates for head preservation treatment, both
hemiarthroplasty and rTSA can provide successful results when applied and performed appropriately. An
increasing body of evidence appears to support consideration of a reverse prosthesis for older patients. A
crucial aspect of hemiarthroplasty success is anatomic healing of the tuberosities around the implant. If the
greater tuberosity displaces or reabsorbs, patients experience significant loss of active motion. The level of
pain, however, is unpredictable. Although beneficial in a reverse prosthesis, anatomic tuberosity position does
not appear to be as crucial. Depending on the definition of “complication,” some researchers have reported a
higher complication rate for the reverse prosthesis. If posttraumatic osteonecrosis develops following head
preservation treatment, conversion to an anatomic shoulder arthroplasty can provide good success in motion
and function, particularly when the tuberosities heal in a relatively anatomic position. However, several
researchers, most recently Moineau and associates, report suboptimal results when the greater tuberosity is
positioned in a substantial amount of varus that necessitates osteotomy. In this scenario, rTSA should be
considered.
Question 50 of 100
Placing a plate too anteriorly against the lateral aspect of the bicipital groove while performing open reduction
and internal fixation (ORIF) of a proximal humerus fracture has an increased risk of what complication?
A. Avascular necrosis
B. Loss of fixation of the fracture
C. Malunion leading to increased retroversion of the articular surface
D. Glenoid arthrosis

PREFERRED RESPONSE: A
DISCUSSION
There are two major arteries that supply the humeral head. One is the ascending branch of the anterior humeral
circumflex artery, which runs up the lateral aspect of the bicipital groove terminating in the arcuate artery.
The other is the posterior humeral circumflex artery, which more recently has been demonstrated to supply a
significant portion of the blood supply to the humeral head. Capsular arteries also play a role in humeral head
perfusion. Care should be taken to preserve all intact arterial supply when performing ORIF, as injury to these
arteries may result in avascular necrosis. In general, the most common complications of locked plating include
loss of reduction with penetration of the joint by the screws, particularly with initial varus positioning of the
humeral head. Placement of the plate in the position described, however, should not have an impact on any of
the other complications noted.

Question 51 of 100
When performing a shoulder hemiarthroplasty for an unreconstructable proximal humerus fracture,
the relationship of the repaired greater tuberosity to the prosthetic humeral head should be
A. 6 mm to 8 mm superior to the top of the humeral head.
B. 6 mm to 8 mm inferior to the top of the humeral head.
C. 1.5 cm inferior to the top of the humeral head.
D. at the same height as the top of the humeral head.

PREFERRED RESPONSE: D
DISCUSSION
The greater tuberosity lies anatomically 6 mm to 8 mm inferior to the top of the humeral head. Normal
proximal humeral anatomy must be recreated when performing a hemiarthroplasty for fracture so as to
minimize the complications associated with the greater tuberosity and maximize functional outcomes.
Tuberosity malunion and nonunion are considered the most common reasons for poor clinical outcomes
following this procedure. Placing the tuberosity too proximal can lead to issues with impingement during
shoulder abduction, and placement too distal can increase the tension on the rotator cuff as it courses over the
prosthetic humeral head.

Question 52 of 100
Figure 1 is the radiograph of a right-hand-dominant 70-year-old woman who arrives at the emergency
department with acute left shoulder pain following a fall down a flight of stairs. She expresses acute diffuse
left shoulder pain and swelling. Prior to her injury, she had fully active, painless shoulder range of motion.
When considering arthroplasty options, which statement regarding the use of hemiarthroplasty or reverse total
shoulder arthroplasty (rTSA) is most accurate?
A. Midterm objective outcomes following hemiarthroplasty are superior.
B. Results following rTSA are less dependent on anatomic tuberosity healing.
C. Use of rTSA is associated with a lower complication rate.
D. Midterm visual analog pain scores following hemiarthroplasty are lower.
PREFERRED RESPONSE: B
DISCUSSION
Fractures of the proximal humerus are now the third most common fracture in patients >60 years of age. This
patient sustained a displaced, commonly described 3-part/4-part proximal humerus fracture. The number of
fracture fragments and angulation, as initially described by Codman and then Neer, does not necessarily help
to predict risk for subsequent AVN. Although the main blood supply to the humeral head historically was
believed to be a branch from the anterior circumflex, adequate perfusion can remain through the posteromedial
calcar following trauma. Hertel and associates reported that the most accurate predictor of ischemia was
whether the length of the metaphyseal head extension for the calcar segment was <8 mm. Locking plates have
provided surgical alternatives to many unstable fracture patterns previously considered ominous. Although
much enthusiasm remains for this implant choice, a relatively high level of complications has been reported
with their use. A multicenter study reported a 14% incidence of intra-articular screw perforation as the most
common complication. When patients are deemed poor candidates for head preservation treatment, both
hemiarthroplasty and rTSA can provide successful results when applied and performed appropriately. An
increasing body of evidence appears to support consideration of a reverse prosthesis for older patients. A
crucial aspect of hemiarthroplasty success is anatomic healing of the tuberosities around the implant. If the
greater tuberosity displaces or reabsorbs, patients experience significant loss of active motion. The level of
pain, however, is unpredictable. Although beneficial in a reverse prosthesis, anatomic tuberosity position does
not appear to be as crucial. Depending on the definition of “complication,” some researchers have reported a
higher complication rate for the reverse prosthesis. If posttraumatic osteonecrosis develops following head
preservation treatment, conversion to an anatomic shoulder arthroplasty can provide good success in motion
and function, particularly when the tuberosities heal in a relatively anatomic position. However, several
researchers, most recently Moineau and associates, report suboptimal results when the greater tuberosity is
positioned in a substantial amount of varus that necessitates osteotomy. In this scenario, rTSA should be
considered.

Question 53 of 100
A 61-year-old right-hand-dominant woman falls down the stairs, resulting in a left anteroinferior dislocation
and noncomminuted greater tuberosity fracture. A closed glenohumeral reduction with intravenous sedation
is performed in the emergency department. After reduction, the greater tuberosity fragment remains displaced
by 2 mm. What is the most appropriate treatment?
A. Open reduction internal fixation with transosseous sutures
B. Arthroscopic fixation using a suture bridge technique
C. Nonsurgical treatment with early passive range of motion
D. Nonsurgical treatment with sling immobilization for 4 weeks

PREFERRED RESPONSE: C
DISCUSSION
Greater tuberosity fractures and rotator cuff tears associated with a traumatic dislocation are more commonly
seen in women >60 years. Greater tuberosity fractures that are displaced <5 mm in the general population and
<3 mm in laborers and professional athletes can be treated successfully without surgery. Early passive range
of motion is important to avoid stiffness.

Question 54 of 100
Figure 1 is the radiograph of a 39-year-old man who has a syncopal episode and fall. After being cleared by
the emergency department, he is referred to your office for left shoulder pain and loss of external rotation.What
is the most appropriate next step in management?

A. Physical therapy
B. CT scan of shoulder
C. Arthroscopic capsular release
D. Arthroscopic Bankart repair
PREFERRED RESPONSE: B
DISCUSSION
The patient has a posterior glenohumeral dislocation, as evidenced by the overlap on the initial radiograph.
While posterior dislocations are rare, they can be overlooked. A CT scan will accurately show the lesion before
proceeding to surgery. If a simple closed reduction is performed acutely and the arm is stable after the
reduction, no further intervention may be needed, and treatment can be successful with a 2-week period of
immobilization for defects involving <30% of the humeral head. However, in this scenario, open reduction is
likely and stabilization may require a modified McLaughlin procedure or other intervention to fill in the
humeral defect. Younger male patients, those with a large humeral head defect, and those with seizure disorder
may be at highest risk for recurrence. For treatment of chronic posterior dislocations, it may be necessary to
perform shoulder arthroplasty to restore stability. Stiffness is attributable to articular incongruity; therefore,
physical therapy and capsular release are inappropriate.

Question 55 of 100
Figure 1 is the radiograph of a 27-year-old man who is involved in a motorcycle collision and sustains a right
femoral and tibial shaft fracture, in addition to the injury shown in Figure 1. All fractures are closed. In addition
to intramedullary nailing of the tibia and femur, appropriate treatment and weight-bearing status of the humeral
shaft fracture should include

A. fracture bracing with full weight bearing.


B. coaptation splinting with non-weight bearing.
C. plate fixation with full weight bearing.
D. plate fixation with non-weight bearing for 4 weeks, followed by full weight bearing.
PREFERRED RESPONSE: D
DISCUSSION
Bell and associates and Tingstad and associates both showed that immediate, full weight bearing through the
upper extremity can be safely allowed for a humeral shaft fracture fixed using a plate and screw construct.
Tingstad and associates showed no difference in malunion or nonunion rate following non-weight bearing or
full weight bearing. Because he is a polytrauma patient, the patient would benefit from operative fixation of
his humerus to expedite recovery and facilitate mobilization. A coaptation splint and a fracture brace would
be appropriate treatment options for a non-polytrauma patient, but in neither case would full weight bearing
generally be allowed immediately following the injury.

Question 56 of 100
A 72-year-old man sustains a displaced four-part fracture of the proximal humerus with head split component
following a fall. A primary shoulder arthroplasty has been recommended for acute management. In counseling
the patient on pros and cons of hemiarthroplasty versus reverse arthroplasty, what statement can be made
based on the available literature?
A. The risk of tuberosity nonunion/malunion appears higher with hemiarthroplasty.
B. Functional outcomes tend to be more consistent with hemiarthroplasty.
C. Forward elevation of reverse shoulder arthroplasty depends on tuberosity union.
D. Active elevation is likely to be better following hemiarthroplasty.

PREFERRED RESPONSE: D
DISCUSSION
As the indications for reverse shoulder arthroplasty have expanded, the role for shoulder hemiarthroplasty
appears to be narrowing. Several recent systematic reviews have evaluated outcomes of shoulder
hemiarthroplasty and reverse shoulder arthroplasty for acute proximal humerus fractures. Their results suggest
that reverse arthroplasty results in superior functional results and comparable elevation, at the expense of
increased complication rates and decreased shoulder rotation. One of the benefits of reverse shoulder
arthroplasty in the setting of fracture is that forward elevation is independent of tuberosity healing and relies
mainly on the deltoid muscle. Active external rotation following a reverse total shoulder for fracture, however,
does appear to depend on successful union of the greater tuberosity. In a randomized controlled trial, the
incidence of tuberosity healing was higher and the incidence of tuberosity resorption was lower in reverse
arthroplasty compared with hemiarthroplasty. Forward elevation following a hemiarthroplasty for fracture
generally follows a bimodal distribution, whereas outcomes following a reverse total shoulder have been more
consistent.

Question 57 of 100
Figure 1 is the radiograph of a 39-year-old man who has a syncopal episode and fall. After being cleared by
the emergency department, he is referred to your office for left shoulder pain and loss of external rotation.What
is the most likely diagnosis?

A. Rotator cuff tear


B. Adhesive capsulitis
C. Brachial plexus injury
D. Posterior shoulder dislocation
PREFERRED RESPONSE: D
DISCUSSION
The patient has a posterior glenohumeral dislocation, as evidenced by the overlap on the initial radiograph.
While posterior dislocations are rare, they can be overlooked. A CT scan will accurately show the lesion before
proceeding to surgery. If a simple closed reduction is performed acutely and the arm is stable after the
reduction, no further intervention may be needed, and treatment can be successful with a 2-week period of
immobilization for defects involving <30% of the humeral head. However, in this scenario, open reduction is
likely and stabilization may require a modified McLaughlin procedure or other intervention to fill in the
humeral defect. Younger male patients, those with a large humeral head defect, and those with seizure disorder
may be at highest risk for recurrence. For treatment of chronic posterior dislocations, it may be necessary to
perform shoulder arthroplasty to restore stability. Stiffness is attributable to articular incongruity; therefore,
physical therapy and capsular release are inappropriate.

Question 58 of 100
A patient sustains a displaced diaphyseal humerus fracture following a motor vehicle accident. Open reduction
internal fixation is indicated due to concomitant lower extremity trauma and is planned through an anterior
approach. Which intramuscular interval is exploited during the deep dissection of the mid-humerus in this
approach?
A. Lateral head of triceps (radial nerve) and brachialis (musculocutaneous nerve)
B. Lateral head of the triceps (radial nerve) and biceps brachii (musculocutaneous nerve)
C. Lateral brachialis (radial nerve) and medial brachialis (musculocutaneous nerve)
D. Brachialis (musculocutaneous nerve) and coracobrachialis (musculocutaneous nerve)
PREFERRED RESPONSE: D
DISCUSSION
The anterior approach to the mid-humerus courses along the lateral margin of the biceps brachii. This muscle
is swept medially allowing exposure of the brachialis. The brachialis has a dual innervation, with the lateral
fibers innervated by the radial nerve and the medial fibers innervated by the musculocutaneous nerve. The
humerus is exposed by splitting this muscle in its midline. The lateral head of the triceps resides in the posterior
compartment of the arm and is not involved in the anterior approach to the humerus. The interval between
brachialis and coracobrachialis is not an internervous plane, as both muscles are supplied by the
musculocutaneous nerve.

Question 59 of 100
Figure 1 is the radiograph of a right-hand-dominant 70-year-old woman who arrives at the emergency
department with acute left shoulder pain following a fall down a flight of stairs. She expresses acute diffuse
left shoulder pain and swelling. Prior to her injury, she had fully active, painless shoulder range of
motion.What is the most common complication following open reduction and locking plate osteosynthesis of
this injury?

A. Intra-articular screw penetration


B. Posttraumatic avascular necrosis (AVN)
C. Surgical neck nonunion
D. Subacromial plate impingement
PREFERRED RESPONSE: A
DISCUSSION
Fractures of the proximal humerus are now the third most common fracture in patients >60 years of age. This
patient sustained a displaced, commonly described 3-part/4-part proximal humerus fracture. The number of
fracture fragments and angulation, as initially described by Codman and then Neer, does not necessarily help
to predict risk for subsequent AVN. Although the main blood supply to the humeral head historically was
believed to be a branch from the anterior circumflex, adequate perfusion can remain through the posteromedial
calcar following trauma. Hertel and associates reported that the most accurate predictor of ischemia was
whether the length of the metaphyseal head extension for the calcar segment was <8 mm. Locking plates have
provided surgical alternatives to many unstable fracture patterns previously considered ominous. Although
much enthusiasm remains for this implant choice, a relatively high level of complications has been reported
with their use. A multicenter study reported a 14% incidence of intra-articular screw perforation as the most
common complication. When patients are deemed poor candidates for head preservation treatment, both
hemiarthroplasty and rTSA can provide successful results when applied and performed appropriately. An
increasing body of evidence appears to support consideration of a reverse prosthesis for older patients. A
crucial aspect of hemiarthroplasty success is anatomic healing of the tuberosities around the implant. If the
greater tuberosity displaces or reabsorbs, patients experience significant loss of active motion. The level of
pain, however, is unpredictable. Although beneficial in a reverse prosthesis, anatomic tuberosity position does
not appear to be as crucial. Depending on the definition of “complication,” some researchers have reported a
higher complication rate for the reverse prosthesis. If posttraumatic osteonecrosis develops following head
preservation treatment, conversion to an anatomic shoulder arthroplasty can provide good success in motion
and function, particularly when the tuberosities heal in a relatively anatomic position. However, several
researchers, most recently Moineau and associates, report suboptimal results when the greater tuberosity is
positioned in a substantial amount of varus that necessitates osteotomy. In this scenario, rTSA should be
considered.

Question 60 of 100
Figures 1 through 3 are the radiograph and select CT images of a 75-year-old smoker with hypertension who
sustains a ground-level fall without loss of consciousness with impact to her left upper extremity 1 week ago.
She reports living independently at home with her husband prior to her fall. What is the most appropriate next
step?

A. Hemiarthroplasty with biceps tenodesis


B. Sling immobilization with subsequent physical therapy
C. Open reduction and internal fixation (ORIF)
D. Reverse total shoulder arthroplasty (rTSA)
PREFERRED RESPONSE: D
DISCUSSION
The radiograph and CT scans indicate a 4-part left proximal humerus fracture with tuberosity comminution.
Based upon her preinjury level of activity and current imaging studies, nonsurgical management is unlikely
to restore her ability to perform activities of daily living, including hygiene care. There has been enthusiasm
among surgeons regarding the use of the reverse shoulder prosthesis as the primary mode of surgical treatment
for certain 3- and 4-part proximal humerus fractures. The main attribute of this implant is its ability to achieve
functional shoulder forward flexion and abduction regardless of tuberosity healing, position, and degree of
comminution. Nevertheless, repair and union of the greater tuberosity fragment during rTSA demonstrate
improved external rotation, clinical outcomes, and patient satisfaction than outcomes achieved after tuberosity
resection, nonunion, or resorption. Based upon this patient’s age and imaging findings, an rTSA would provide
pain relief and improved function with complication rates similar to those associated with hemiarthroplasty.
ORIF would not be a viable option because of the high probability for a dysvascular head, increased risk for
nonunion, and potential for revision surgery, including arthroplasty. Hemiarthroplasty for 4-part proximal
humerus fractures remains a viable option for patients <70 years of age with minimal tuberosity comminution
and an intact rotator cuff who can comply with a postsurgical rehabilitation program. Most studies indicate
significant pain relief with this modality, with significant variation in functional outcomes. In this clinical
scenario, the patient’s injury may not be best served with hemiarthroplasty because of uncertainty regarding
functional outcome.

Question 61 of 100
A 67-year-old man with right shoulder osteoarthritis (OA) remains symptomatic despite a course of
nonsurgical treatment. A CT scan of the shoulder shows eccentric posterior glenoid wear with 10° of
retroversion. What is the appropriate management of this glenoid bone loss during surgery for an anatomic
total shoulder arthroplasty?
A. In situ glenoid component implantation
B. Hemiarthroplasty
C. Eccentric reaming of glenoid
D. Posterior glenoid bone graft

PREFERRED RESPONSE: C
DISCUSSION
Total shoulder arthroplasty (TSA) is superior to hemiarthroplasty for primary OA. The most common
complication of TSA is glenoid loosening and malposition, which are common causes of glenoid failure.
Glenoid malposition decreases the glenohumeral contact area and subsequently increases contact pressures.
Altering the stem version to accommodate glenoid retroversion does not appropriately address soft-tissue
balancing. A retroversion of <12° to 15° can be corrected with eccentric reaming without excessively
compromising glenoid bone stock and risking glenoid vault penetration by the glenoid component. Posterior
glenoid bone grafting may be considered for glenoid retroversion >15°.

Question 62 of 100
A healthy 65-year-old woman undergoes anatomic total shoulder arthroplasty to address osteoarthritis (OA).
The surgery is uncomplicated. What is the most common indication for future revision?
A. Deep infection
B. Periprosthetic fracture
C. Glenoid component loosening
D. Rotator cuff tear

PREFERRED RESPONSE: C
DISCUSSION
The most common reason for revision surgery following unconstrained shoulder arthroplasty for
glenohumeral OA is loosening of an implant. In most studies that distinguish glenoid from humeral loosening,
it appears the glenoid is the problem. Comprehensive systematic reviews have found that radiographic glenoid
loosening can comprise nearly 30% to 40% of all complications following shoulder arthroplasty for non-
inflammatory arthritis. Infections, periprosthetic fractures, and rotator cuff tears are uncommon. In the
population-based study by Matsen and associates, 10% of the revisions were performed for loosening versus
7% for infection and 7% for rotator cuff tearing.

Question 63 of 100
A 25-year-old minor league baseball pitcher has posteromedial elbow pain during the deceleration phase of
his motion. Examination reveals a mild flexion contracture of the elbow, moderate tenderness over the medial
olecranon, mild tenderness over the radiocapitellar joint and medial collateral ligament, and a grossly stable
valgus stress test. Figure 1 is a coronal MR arthrogram of his elbow. What is the most likely diagnosis?

A. Incomplete tear of the medial collateral ligament


B. Incomplete tear of the lateral collateral ligament
C. Valgus extension overload (VEO)
D. Osteochondritis dissecans of the capitellum
PREFERRED RESPONSE: C
DISCUSSION
VEO is a condition most frequently seen in repetitive overhead throwing athletes. It represents a combination
of pathologies resultant from tensile stress on the medial collateral ligament and common flexor pronator
origin, lateral compressive force across the radiocapitellar joint, and shear stress between the medial tip of the
olecranon and olecranon fossa. The deceleration and follow-through phases of the throwing motion are when
contact between the olecranon tip and fossa are at their highest, often resulting in osteophyte formation that
can both block terminal extension and cause local tenderness. While compromise of the medial collateral
ligament (MCL) and osteochondral injury of the radiocapitellar joint can occur in VEO, these findings are part
of the larger disease process. In this patient, the MR arthrogram reveals a grossly intact MCL and no evidence
of radiocapitellar degeneration. Compromise of the lateral ulnar collateral ligament is not a component of
VEO and is not suggested by the primary site of pain in the question.

Question 64 of 100
A 19-year-old, right-hand-dominant collegiate baseball pitcher reports a 4-month history of right shoulder
pain after a throwing activity. He localizes the pain primarily to the posterior aspect of his shoulder and
describes the type of pain as an aching sensation. He has been involved with strength and conditioning with
his team but denies any specific therapy other than the application of ice after throwing and use of occasional
over-the-counter anti-inflammatory drugs, neither of which has provided relief. He denies any specific
traumatic event or previous history of shoulder problems. His pitching coach has noted a slight decrease in his
throwing velocity during the last 2 months. The patient fails nonsurgical treatment and undergoes shoulder
arthroscopy. At the time of surgery, the area marked by the asterisk in Figure 1 is visualized from the
posterolateral portal. This anatomic structure impinges on which other structure during late cocking of the
throwing phase?

A. Biceps tendon
B. Posterior band of the inferior glenohumeral ligament
C. Hill-Sachs lesion
D. Undersurface of the supraspinatus and infraspinatus tendons

PREFERRED RESPONSE: D
DISCUSSION
This patient’s clinical presentation is consistent with internal impingement accompanied by glenohumeral
internal rotation deficit (GIRD). Although throwers may have increased external rotation, their overall arc of
motion should be the same as on the nonthrowing side. In comparison, patients with GIRD experience a
marked decrease in arc of motion, particularly in internal rotation. Internal impingement represents a spectrum
of findings that can include superior and posterior labral tears, undersurface (articular-sided) tearing of the
posterior supraspinatus, posterior glenoid wear, and scar formation of the posterior capsule. Myers and
associates demonstrated internal impingement is associated with GIRD, although the latter by itself may be
asymptomatic and perhaps a sports-specific adaptation. However, posterior capsular tightness can lead to
posterosuperior translation of the humerus during throwing, leading to these injuries. Internal impingement is
common among overhead throwing athletes and occurs during the late cocking and early acceleration phases
of throwing. Humeral migration during the abducted/externally rotated throwing position results in abutment
of the greater tuberosity against the posterosuperior glenoid labrum, which impinges the rotator cuff (Paley
and associates). Pain is often posterior, but symptoms can be vague. Patients may have examination findings
consistent with rotator cuff weakness and superior labrum anterior to posterior (SLAP)/biceps involvement.
Radiographic findings can be negative, although a Bennett lesion involving hypertrophy and mineralization
of the posterior capsular injury may be seen (Wright and Paletta). A CT scan may show glenoid retroversion
(Crockett and associates), whereas MR imaging should be reviewed for a possible partial articular-sided
rotator cuff tear, SLAP tear, or increased signal in the posterosuperior labrum, or greater tuberosity. Treatment
of this condition should be the focus on therapy, and most cases can be treated nonsurgically. Stretching aimed
at the posterior capsule (ie, sleeper stretch) has been reported as effective (Tyler and associates, Lintner and
associates). Burkhart and associates also demonstrated that posterior capsular stretching can help to prevent
throwing injuries. Because cuff pathology may be present, physical therapy also should include rotator
strengthening, scapular stabilization, and addressing of issues related to throwing mechanics (Drakos and
associates). Kibler and associates published a comprehensive rehabilitation guideline. Surgical intervention is
reserved for those who fail 6 months of nonsurgical treatment and is directed by intra-articular pathology
(debridement vs repair of the rotator cuff and labrum) (Braun and associates).

Question 65 of 100
Figures 1 through 4 are the radiograph and MRI scans of a 70-year-old woman who has a 10-year history of
worsening shoulder pain. She has had multiple corticosteroids, several rounds of physical therapy, and
continues to take nonsteroidal anti-inflammatory medications. She has pain with all activities and has to use
the contralateral hand to aid in elevation of the arm. The pain prevents her from sleeping. Her active forward
elevation is 40°; her passive forward elevation is 160° with a positive lag sign. Her active external rotation
with the arm at the side is 10°; her passive external rotation is 40° with a positive lag sign. Her hornblower’s
sign is negative. What would be the most effective treatment option for this patient?

A. Arthroscopic rotator cuff repair


B. Anatomic total shoulder arthroplasty
C. Reverse total shoulder arthroplasty with latissimus transfer
D. Reverse total shoulder arthroplasty
PREFERRED RESPONSE: D
DISCUSSION
This patient has failed nonsurgical measures and continues to demonstrate pseudoparalysis. The images show
proximal humeral migration with acetabularization of the acromion. The teres minor is intact and the
hornblower’s sign is negative. This patient would benefit most from a reverse total shoulder arthroplasty. If
there was severe atrophy or absence of teres minor and/or if the hornblower’s sign was positive, then the best
choice for treatment to restore external rotation would be reverse total shoulder arthroplasty with a latissimus
transfer. Ten-year follow-up following reverse total shoulder arthroplasty for the treatment of a massive rotator
cuff tear in the setting of arthritis demonstrated >90% survivorship. All patients demonstrated improved range
of motion and pain relief, as well as improved patient-reported outcomes scores. The key to restoration of
function, especially external rotation, is the teres minor. Indications for the addition of a latissimus transfer to
reverse total shoulder arthroplasty include rotator cuff tears in which the teres minor is absent or atrophic (as
all remaining external rotators are absent). Patients who do not regain external rotation control will have a
difficult time performing activities of daily living such as bringing their hand to their mouth. The rotator cuff
in the MRI scan demonstrates significant retraction and fatty atrophy/infiltration, making repair of the tendon
a non-viable option. An unconstrained total shoulder arthroplasty requires an intact rotator cuff, or a repairable
tendon, to minimize edge-loading of the glenoid component and potential loosening.

Question 66 of 100
A 37-year-old recreational athlete has osteoarthritis of the glenohumeral joint. He has failed nonsurgical
measures and is interested in surgical intervention but would like to avoid arthroplasty. When performing
shoulder arthroscopy for glenohumeral arthritis, which radiographic parameter is most predictive of clinical
failure?
A. Unipolar arthritis
B. >3 mm of glenohumeral joint space
C. Walch B2 glenoid morphology
D. Small inferior humeral osteophyte

PREFERRED RESPONSE: C
DISCUSSION
Multiple studies have evaluated the utility of arthroscopy in the treatment of shoulder arthritis. Despite
differing levels of success, a few common characteristics have been shown to lead to a higher probability of
clinical failure. Mitchell and associates showed that shoulders with less joint space (1.3 mm vs 2.6 mm) and
Walch type B2 and C glenoids were significantly more likely to fail than were Walch types A1, A2, and B1.
Additionally, older patients (age >50 years) tended to have worse outcomes. Skelley and associates found that
isolated capsular release and debridement had a high failure rate (conversion to total shoulder arthroplasty in
42% within 9 months) and postulated that patients undergoing concomitant procedures, such as biceps
tenodesis, may fare better. Van Theil and associates found significant risk factors for failure included the
presence of grade 4 bipolar disease, joint space <2 mm, and the presence of large osteophytes. They had a
22% conversion to total shoulder arthroplasty at 10.1 months.

Question 67 of 100
A 19-year-old, right-hand-dominant collegiate baseball pitcher reports a 4-month history of right shoulder
pain after a throwing activity. He localizes the pain primarily to the posterior aspect of his shoulder and
describes the type of pain as an aching sensation. He has been involved with strength and conditioning with
his team but denies any specific therapy other than the application of ice after throwing and use of occasional
over-the-counter anti-inflammatory drugs, neither of which has provided relief. He denies any specific
traumatic event or previous history of shoulder problems. His pitching coach has noted a slight decrease in his
throwing velocity during the last 2 months.Which image seen during arthroscopic treatment is most likely
associated with this patient's condition?

PREFERRED RESPONSE: A
DISCUSSION
This patient’s clinical presentation is consistent with internal impingement accompanied by glenohumeral
internal rotation deficit (GIRD). Although throwers may have increased external rotation, their overall arc of
motion should be the same as on the nonthrowing side. In comparison, patients with GIRD experience a
marked decrease in arc of motion, particularly in internal rotation. Internal impingement represents a spectrum
of findings that can include superior and posterior labral tears, undersurface (articular-sided) tearing of the
posterior supraspinatus, posterior glenoid wear, and scar formation of the posterior capsule. Myers and
associates demonstrated internal impingement is associated with GIRD, although the latter by itself may be
asymptomatic and perhaps a sports-specific adaptation. However, posterior capsular tightness can lead to
posterosuperior translation of the humerus during throwing, leading to these injuries. Internal impingement is
common among overhead throwing athletes and occurs during the late cocking and early acceleration phases
of throwing. Humeral migration during the abducted/externally rotated throwing position results in abutment
of the greater tuberosity against the posterosuperior glenoid labrum, which impinges the rotator cuff (Paley
and associates). Pain is often posterior, but symptoms can be vague. Patients may have examination findings
consistent with rotator cuff weakness and superior labrum anterior to posterior (SLAP)/biceps involvement.
Radiographic findings can be negative, although a Bennett lesion involving hypertrophy and mineralization
of the posterior capsular injury may be seen (Wright and Paletta). A CT scan may show glenoid retroversion
(Crockett and associates), whereas MR imaging should be reviewed for a possible partial articular-sided
rotator cuff tear, SLAP tear, or increased signal in the posterosuperior labrum, or greater tuberosity. Treatment
of this condition should be the focus on therapy, and most cases can be treated nonsurgically. Stretching aimed
at the posterior capsule (ie, sleeper stretch) has been reported as effective (Tyler and associates, Lintner and
associates). Burkhart and associates also demonstrated that posterior capsular stretching can help to prevent
throwing injuries. Because cuff pathology may be present, physical therapy also should include rotator
strengthening, scapular stabilization, and addressing of issues related to throwing mechanics (Drakos and
associates). Kibler and associates published a comprehensive rehabilitation guideline. Surgical intervention is
reserved for those who fail 6 months of nonsurgical treatment and is directed by intra-articular pathology
(debridement vs repair of the rotator cuff and labrum) (Braun and associates).
Question 68 of 100
Figures 1 through 4 are the radiographs and CT scans of a 78-year-old right-hand dominant man with a recent-
onset painful left shoulder and limited range of motion. He was reaching overhead and felt a pop, which
resulted in severe pain and dysfunction. The patient underwent an anatomic total shoulder arthroplasty 5 years
prior for glenohumeral osteoarthritis. A select axial CT image from before the index surgery is seen in Figure
4. The patient has not had any fevers or systemic symptoms of infection. ESR, CRP, and CBC levels remain
normal. What preoperative factors are most predictive of the complication experienced by this patient?

A. Patient age at the time of index surgical procedure


B. Presence of a small repairable full-thickness rotator cuff tear
C. Use of pegged all-polyethylene cemented glenoid component
D. Severe baseline glenoid erosion and posterior humeral subluxation
PREFERRED RESPONSE: D
DISCUSSION
This patient has an anatomic total shoulder arthroplasty with a completely dislocated glenoid component. The
glenoid component can be seen in the posterior axillary pouch, as evidenced by the radiographic marker seen
best on the axial cut CT. Preoperative factors that influence the outcome of an anatomic total shoulder
arthroplasty resulting in the need for revision surgery for failed glenoid component include preoperative fixed
posterior humeral head subluxation and moderate to severe eccentric glenoid erosion. Walch and associates
describe three patterns of glenoid component migration including superior tilting, subsidence, and posterior
tilting. Superior tilting was associated with implant position and rotator cuff integrity. Subsidence was
associated with aggressive reaming for correction of glenoid version, which does not maintain the subchondral
bone support. Posterior subluxation was associated with glenoid erosion and posterior humeral head
subluxation.

Question 69 of 100
Placement of the most distal interlocking screw seen in the radiographs provided in Figures 1 and 2 would
most likely result in what clinical examination finding?

A. Weakness of elbow flexion


B. Inability to extend thumb interphalangeal (IP) joint
C. Decreased sensation over radial/volar forearm
D. Decreased sensation of small and ring fingers
PREFERRED RESPONSE: C
DISCUSSION
Blunt dissection and soft-tissue protection is warranted with distal interlocking screw placement following
humeral intramedullary nailing. The most distal locking screw in this intramedullary nail construct was placed
from anterior to posterior, passing through the distal portion of the biceps and brachialis muscle bellies. The
musculocutaneous nerve which continues as the sensory lateral antebrachial cutaneous (LABC) nerve is at
risk, as it lies between these two muscles. Injury to the LABC nerve results in decreased sensation over the
radial volar aspect of the forearm. Malrotation of the nail, producing a more anteromedial starting point for
the anterior-to-posterior screw, can lead to a path that intersects with the median nerve and brachial artery.
More commonly, the median nerve can be injured with overpenetration of the medial cortex with a lateral-to-
medial directed screw. Median nerve injury would affect innervations of the flexor digitorum superficialis and
profundus to the index finger (among other motors). Although the dissection violates the muscle belly of these
two elbow flexors, measurable weakness in elbow flexion is not typically seen. The radial nerve has already
provided function to triceps (elbow extension) proximal to this level and lies sufficiently lateral to be more of
a concern with a lateral-to-medial screw placement (thumb IP extension). The ulnar nerve (decreased sensation
of small and ring fingers) is further medial at this level and would similarly be at risk with a lateral-to-medial
interlocking screw.
Question 70 of 100
A 72-year-old active man has shoulder pain after undergoing an explantation of an anatomic shoulder
arthroplasty 6 months prior with an antibiotic cement spacer placed. The patient has 60° of forward flexion,
40° of external rotation, and a positive belly press with limited internal rotation. A recent work-up for
continued infection is negative, and a follow-up MRI reveals grade 2 atrophy of the supraspinatus and grade
3 atrophy of the subscapularis with tendon retraction to the glenoid rim. What is the best next step in definitive
management?
A. Revision anatomic total shoulder arthroplasty
B. Reverse total shoulder arthroplasty
C. Hemiarthroplasty with latissimus dorsi transfer
D. Resection arthoplasty

PREFERRED RESPONSE: B
DISCUSSION
This patient has a previously failed total shoulder arthroplasty for which he underwent placement of an
antibiotic spacer, and now has continued shoulder pain. The recent MRI demonstrates a likely
irreparable subscapularis tendon, making revision with an anatomic shoulder arthroplasty contraindicated.
Use of a hemiarthroplasty is unlikely to restore function in this older patient with underlying rotator cuff
disease, though it may be helpful for pain relief. Furthermore, a latissimus dorsi transfer is also
contraindicated in the setting of a chronic subscapularis tear. A reverse shoulder arthroplasty offers the most
reliable clinical outcome. Given that the preoperative infection work-up was negative, resection arthroplasty
is not indicated for this otherwise active patient.

Question 71 of 100
Figures 1 and 2 are the radiograph and axial CT scan of a 75-year-old woman with diffuse superior shoulder
pain 5 months after an uneventful reverse shoulder arthroplasty. She denies trauma, but felt a "pop" when
reaching overhead. She had initially done well postoperatively. On physical examination, she has decreased
active forward flexion with pain and diffuse superior tenderness along the scapular spine and acromion. There
are no signs or symptoms of infection. What is the best next step in management?
A. Application of bone stimulator
B. Open reduction and internal fixation (ORIF)
C. Physical therapy for deltoid strengthening
D. Sling immobilization for 6 weeks
PREFERRED RESPONSE: D
DISCUSSION
The radiograph shows a well-positioned reverse total shoulder and the axial CT image demonstrates a
minimally displaced fracture through the midacromion. Acromial fracture is a concerning and not uncommon
complication that can have devastating effects after reverse shoulder arthroplasty, as function is highly
dependent on the deltoid muscle. Fracture can occur at the scapular spine, which may be related to placement
of peripheral baseplate screws. Acromial fracture has been reported to occur as early as 1 month
postoperatively and as late as 8 years postoperatively. The reported incidence is <8%, and multiple authors
note decreases in shoulder elevation and shoulder outcome scores compared with that in patients with reverse
shoulder arthroplasty without a fracture. Patients with a fracture can complain of superior shoulder pain that
may radiate to the deltoid area and usually relate an acute onset of pain or loss of function after initially good
clinical progress. The diagnosis can be difficult to determine and is missed on plain radiographs in up to 20%
of cases. CT scans are helpful. Treatment is typically nonoperative with sling immobilization for 6 weeks and
then advancing activities as tolerated. A bone stimulator and ORIF would not be indicated for a minimally
displaced acute acromion fracture. Continued physical therapy with strengthening would also be
contraindicated.

Question 72 of 100
Figures 1 and 2 are the radiographs of a 30-year old recreational polo player who sustained an injury to his
right shoulder following a fall from a horse. He denies any prior injuries to the shoulder. He reports pain in
the superior aspect of the shoulder and has an abrasion over the lateral acromion. Which anatomic structure is
most important for maintaining the anterior-posterior stability of the injured joint?

A. Conoid ligament
B. Trapezoid ligament
C. Inferior acromioclavicular (AC) ligament
D. Posterosuperior AC ligament
PREFERRED RESPONSE: D
DISCUSSION
The patient has radiographic evidence of a grade III AC joint separation. Treatment of grade III AC separations
remains controversial. In a grade III AC separation, both the AC joint capsule, as well as the coracoclavicular
ligaments (conoid and trapezoid) are disrupted. The coracoclavicular ligaments primarily provide
superior/inferior stability to the clavicle, and the AC joint capsule/ligament complex provides anterior-
posterior stability. Within this capsule complex, biomechanical studies have shown that the posterosuperior
portion is most crucial for maintaining anterior-posterior stability of the AC joint and should be preserved
during AC joint resection procedures. The inferior AC capsule/ligament does not play a significant role in AC
joint horizontal stability.

Question 73 of 100
Figure 1 is the radiograph of a 54-year-old man who has increasing weakness and numbness in his lateral arm.
No prior surgery or injury is reported. What is the most appropriate next diagnostic test?

A. MRI of the shoulder


B. MRI of the cervical spine
C. CT scan of the chest
D. Radiograph of the chest
PREFERRED RESPONSE: B
DISCUSSION
The radiograph reveals a Charcot neuropathic shoulder. The atraumatic destruction of the humeral head is
concerning for a neuropathic etiology and warrants MR imaging of the cervical spine to evaluate for the
presence of a syrinx. Shoulder arthroplasty in the setting of a neuropathic joint is challenging given the local
bone and soft-tissue loss, in addition to the loss of protective sensation. The scant literature on the use of
shoulder arthroplasty in these challenging patients reports an improvement in pain but only modest
improvements in shoulder function.
Question 74 of 100
A 17-year-old high school football player sustains a traumatic anterior shoulder dislocation, resulting in a
small bony Bankart lesion and small Hill-Sachs lesion. The patient undergoes an arthroscopic Bankart repair
with incorporation of the bone fragment and returns to play football the following year. He has a recurrent
dislocation at football practice but decides to finish the football season before considering additional treatment.
He sustains nine additional dislocations, with the last dislocation occurring while sleeping. The patient has
eroded one-third of the inferior glenoid surface area. What is the most appropriate surgical treatment?
A. Revision arthroscopic Bankart repair with capsular shift
B. Open Bankart repair with capsular shift
C. Repair of infraspinatus tendon into the Hill-Sachs defect (remplissage procedure)
D. Coracoid transfer to the glenoid (Latarjet procedure)

PREFERRED RESPONSE: D
DISCUSSION
A failed bony Bankart repair with multiple dislocations can further erode the anteroinferior glenoid, changing
the sagittal morphology of the glenoid into an “inverted pear.” Quantitative bone loss is best evaluated by CT
scan with 3-D reconstructions and subtraction of the humeral head. MRI and ultrasonography can assist in
evaluating soft-tissue injury, but they are not as helpful in determining bone loss compared with a CT scan.
An arthrogram alone is not sufficient to evaluate bone loss. Bone loss >30% necessitates glenoid augmentation
with either a Latarjet procedure or iliac crest bone grafting. A revision arthroscopic or open Bankart repair
with capsular shift or remplissage do not address bone loss. The Latarjet procedure can effectively restore
stability with glenoid bone loss and after failed stabilizing procedures. Patients with pain before surgery are
more likely to have pain after surgery. Age and activity level are lesser influences on satisfaction.

Question 75 of 100
What phase of overhead throwing puts the rotator cuff at most risk of injury from internal impingement?
A. Wind up
B. Late cocking
C. Deceleration
D. Follow through

PREFERRED RESPONSE: D
DISCUSSION
Internal impingement occurs when there is repetitive contact of the posterior superior aspect of the glenoid
with the humeral head causing damage to the undersurface of the supraspinatus and anterior aspect of the
infraspinatus tendons, as well as posterior superior glenoid labrum. This occurs when the arm is in maximum
abduction and external rotation such as during the late cocking phase of the normal throwing motion. The 6
phases of throwing are wind up, early cocking, late cocking, deceleration, and follow through. When the arm
is repeatedly placed in the abducted externally rotated position, the anterior capsule can become lax and
posterior capsular contractures can develop. When there are kinetic chain abnormalities such as scapular
internal rotation or muscle fatigue, there is exacerbation of abnormal anterior humeral head translation and
increased contact of the rotator cuff on the posterior glenoid rim, with concomitant increased risk of injury
and symptoms.

Question 76 of 100
A 65-year-old man undergoes an uneventful left total shoulder arthroplasty for primary osteoarthritis using a
lesser tuberosity osteotomy. At his 6-week postoperative visit, he is progressing well with physical therapy.
Two months after surgery, he slips and falls in his driveway. His subsequent visits over the next 8 weeks with
therapy are fraught with marked deterioration of his active motion and the inability to reach his lower back.
A subsequent radiograph and axial CT view of his shoulder are shown in Figures 1 and 2. What is the best
next step in management?

A. Sling immobilization and continued formal therapy


B. Open excision of heterotopic bone formation
C. Open reduction internal fixation of lesser tuberosity
D. Conversion to reverse shoulder arthroplasty
PREFERRED RESPONSE: D
DISCUSSION
Anatomic total shoulder arthroplasty remains the most utilized surgical modality for patients with primary
osteoarthritis of the shoulder. Recently, the lesser tuberosity osteotomy has generated enthusiasm as a
theoretical avenue to improve subscapularis function postoperatively. With modern repair techniques, this
approach has a reported high union rate, making it a comparable alternative with the traditional subscapularis
peel or tenotomy. However, loss of fixation and subscapularis failure may still occur. The patient’s radiograph
reveals a medially displaced lesser tuberosity with extension of a fracture line to the lateral cortex of the
proximal humerus. As a result, the patient now has subscapularis insufficiency that has affected his active
range of motion, as evidence by his inability to reach his lower back. Owing to the timing following the injury
to presentation, conversion to a reverse shoulder arthroplasty would most reliably maximize the patient’s
outcome. In the acute setting, an attempt at operative repair of the failed lesser tuberosity may be considered.
Sling immobilization would not be recommended. One of the benefits of an osteotomy is that plain radiographs
are often more than adequate to confirm the diagnosis of subscapularis failure secondary to loss of tuberosity
fixation.

Question 77 of 100
A 23-year-old left-hand dominant professional football player sustains a left shoulder injury after being
tackled and lands directly on his shoulder 1 month ago. The patient was diagnosed with a Rockwood type 2
acromioclavicular separation. Following physical therapy, his symptoms have improved. He has good scapular
control and shoulder strength. What physical examination test would help determine the contribution of the
acromioclavicular joint injury to his residual symptoms?
A. O'Brien test
B. Hawkins-Kennedy test
C. Dynamic labral shear test (DLST)
D. Upper cut test
PREFERRED RESPONSE: A
DISCUSSION
The clinical scenario describes an athlete who is recovering from a type 2 acromioclavicular joint separation.
The goal of this question is to stress the importance of the physical examination to guide treatment decisions,
as well as recovery. It is important to recognize which factors can aid in decision making especially with type
type 2 acromioclavicular joint separation injuries as the data are still not clear as to who would best be served
with surgical versus nonsurgical management. The active compression test as described by O’Brien and
associates in 1998 was equally as effective at assessing the acromioclavicular joint as it was for assessment of
the integrity of the superior labrum. The Hawkins-Kennedy test has demonstrated utility in the diagnosis of
rotator cuff impingement, wherein the greater tuberosity comes into contact with the coracoacromial ligament.
The DLST has been described for the diagnosis of superior labral anterior-posterior (SLAP) tears, wherein the
patient reports pain and a click felt with movement of the shoulder through an arc of abduction with the
shoulder externally rotated. The upper cut test has been described in the setting of biceps tendinopathy and
SLAP tears.

Question 78 of 100
Stemless shoulder arthroplasty prostheses have recently been suggested as an alternative to traditional
stemmed replacement. Advantages of the stemless surgical technique would include
A. better glenoid exposure than with stemmed prosthesis.
B. reliable use in four-part proximal humerus fracture surgery.
C. use in proximal humeral malunion without need for osteotomy.
D. improved long-term survivorship profile.

PREFERRED RESPONSE: C
DISCUSSION
Glenoid exposure, while better than with surface replacements, is not improved over traditional stemmed
replacements. Metaphyseal comminution would make it unlikely that a stemless implant could be used in most
four-part fractures. Stemless replacement does have the unique advantage of allowing placement of a
prosthesis with a malunion without an osteotomy, as the prosthesis is not constrained by the position of the
stem. While early results are encouraging, there is no long-term data to suggest that survivorship is increased
with stemless arthroplasty.

Question 79 of 100
A 73-year old woman reports activity-related pain in the right shoulder. She had undergone surgery on her
shoulder following a fall at home. A select radiograph is given in Figure 1. The radiographic changes seen
along the glenoid neck would most likely have been minimized through the use of what implant-related
technique?
A. More valgus-angled humeral stem
B. More distal placement of glenoid baseplate
C. Decreased lateral offset
D. Increased number of baseplate locking screws
PREFERRED RESPONSE: B
DISCUSSION
In a reverse shoulder arthroplasty, multiple designs with varying amounts of valgus neck-shaft angles have
been developed since the original 155° of the neck-shaft angle Grammont prosthesis design. Regardless of
neck-shaft angle utilized, average forward elevation flexion improvement ranges from 78° to 131°, with no
difference in dislocation rates and range of motion. A 135° neck-shaft angle has been shown to have less
scapular notching. The radiograph demonstrates a commonly utilized reverse total shoulder replacement with
a 135°-angled humeral stem. The baseplate has not been placed at the inferior-most aspect of the glenoid with
the result being that the inferior aspect of the glenosphere lies superior to the glenoid neck. While the clinical
implications of scapular notching were previously controversial, it has more recently been shown that
advanced scapular notching is associated with a decrease in functional outcomes and possibly baseplate
loosening. Strategies for reducing the prevalence of notching have been shown through clinical, computer
simulation, and biomechanical studies. A more valgus stem, increased superior tilt, and a more medialized
offset have been shown to increase the risk for scapular notching. The number of screws can affect initial
baseplate stability but has not been shown to have an effect on scapular notching. In this case, more distal
placement of the baseplate (or use of a larger or more laterally offset glenosphere) would have decreased the
adduction deficit and reduced impingement of the polyethylene onto the lateral scapula.

Question 80 of 100
Figures 1 and 2 are the radiographs of a 69-year-old, left-hand-dominant retired man with left shoulder pain.
The pain has been present for several years. He worked in construction but retired 3 years ago. He now reports
pain interfering with activities around the house but denies recent trauma or prior shoulder surgery. He has
tried nonsteroidal anti-inflammatory drugs, but these do not provide complete relief. He demonstrates pain
and crepitus with active and passive shoulder motion. He can actively forward flex to 100° and external rotate
to 30°. Rotator cuff testing reveals 5/5 strength and he is neurovascularly intact. After discussion regarding
surgical and nonsurgical treatment options, the patient wishes to proceed with surgical intervention. He has
done online research and has questions about which procedure will produce the best outcome. Based on the
current literature, what is the most appropriate surgical procedure?

A. Arthroscopic glenohumeral debridement with biceps tenotomy


B. Hemiarthroplasty
C. Total shoulder arthroplasty (TSA)
D. Reverse TSA (rTSA)
PREFERRED RESPONSE: C
DISCUSSION
The examination and radiograph findings are consistent with glenohumeral osteoarthritis (OA), which is now
interfering with this patient’s daily activities. Regarding surgical treatment for glenohumeral arthritis, several
studies have shown that TSA is associated with better functional outcomes than hemiarthroplasty. A recent
meta-analysis comparing outcomes between TSA and hemiarthroplasty revealed better function after TSA
regarding pain, University of California-Los Angeles Shoulder Scores, and postsurgical forward elevation at
a minimum of 2 years (Bryant and associates). In addition, Gartsman and associates showed that TSA resulted
in better pain relief, function, strength, and patient satisfaction than hemiarthroplasty at 3-year follow-up. The
patient in this scenario exhibits good rotator cuff strength on examination; therefore, arthroscopy with
debridement or rTSA used in the setting of a massive cuff tear would not be the best answer. Several studies
have similarly shown good results with TSA in the setting of inflammatory and rheumatoid arthritis (Thomas
and associates, Jolles and associates). Posterior glenoid wear is a common pattern in OA and not a
contraindication (Walch and associates). However, patients with a brachial plexus root avulsion or
preganglionic injury resulting in a flail arm are not candidates for TSA because of the poor prognosis for
recovery of motor and sensory deficits. Inflammatory arthritis characteristically results in concentric glenoid
wear and not the eccentric posterior erosion seen in OA. This concentric wear results in medialization of the
glenohumeral joint line. This wear pattern can lead to severe erosion, making it difficult to achieve glenoid
resurfacing. Eccentric posterior glenoid wear would lead to posterior humeral head subluxation, which is not
as common in inflammatory arthritis. In addition, inferior humeral head osteophytes are seen in OA. Walch
and associates have developed a classification system describing glenoid wear patterns. During TSA, exposure
to the glenohumeral joint involves subscapularis management, tendon peel, or lesser tuberosity osteotomy. As
a result, early postsurgical rehabilitation limits passive external rotation and active internal rotation, typically
for 6 weeks, to protect the subscapularis repair. The patient in this scenario likely eccentrically contracted his
subscapularis, resulting in repair failure. Upon clinical examination, internal rotation weakness, increased
passive external rotation, and abnormal belly press or lift-off test results can be expected. One study revealed
that rupture of the subscapularis was seen in all anterior dislocations following TSA (Wirth and Rockwood).

Question 81 of 100
A 75-year-old man sustains an anterior dislocation of his reverse total shoulder arthroplasty. What activity
places the arm in the position most commonly associated with reverse total shoulder dislocation?
A. Scratching the opposite shoulder
B. Pushing off ipsilateral chair armrest while standing up
C. Tying shoelaces on the contralateral foot
D. Reaching up to comb hair

PREFERRED RESPONSE: B
DISCUSSION
Proper soft-tissue tension is critical to prevent instability of a reverse total shoulder implanted with the
deltopectoral approach; dislocation of the prosthesis is exceedingly rare if the superior approach is employed.
The arm position implicated in reverse total shoulder instability is extension, adduction, and internal rotation,
such as pushing out of a chair. The other positions described do not involve extension of the shoulder.

Question 82 of 100
Figures 1 and 2 are the radiographs of a 69-year-old, left-hand-dominant retired man with left shoulder pain.
The pain has been present for several years. He worked in construction but retired 3 years ago. He now reports
pain interfering with activities around the house but denies recent trauma or prior shoulder surgery. He has
tried nonsteroidal anti-inflammatory drugs, but these do not provide complete relief. He demonstrates pain
and crepitus with active and passive shoulder motion. He can actively forward flex to 100° and external rotate
to 30°. Rotator cuff testing reveals 5/5 strength and he is neurovascularly intact. During the patient's
presurgical history and physical visit, he reports to the nurse that he has a history of rheumatoid arthritis which
is being managed by his primary care physician. With this new information in hand, which finding is most
commonly seen on imaging during presurgical planning?
A. Glenoid medialization
B. Posterior glenoid wear
C. Posterior subluxation of the humeral head
D. Inferior osteophytes at the humeral head
PREFERRED RESPONSE: A
DISCUSSION
The examination and radiograph findings are consistent with glenohumeral osteoarthritis (OA), which is now
interfering with this patient’s daily activities. Regarding surgical treatment for glenohumeral arthritis, several
studies have shown that TSA is associated with better functional outcomes than hemiarthroplasty. A recent
meta-analysis comparing outcomes between TSA and hemiarthroplasty revealed better function after TSA
regarding pain, University of California-Los Angeles Shoulder Scores, and postsurgical forward elevation at
a minimum of 2 years (Bryant and associates). In addition, Gartsman and associates showed that TSA resulted
in better pain relief, function, strength, and patient satisfaction than hemiarthroplasty at 3-year follow-up. The
patient in this scenario exhibits good rotator cuff strength on examination; therefore, arthroscopy with
debridement or rTSA used in the setting of a massive cuff tear would not be the best answer. Several studies
have similarly shown good results with TSA in the setting of inflammatory and rheumatoid arthritis (Thomas
and associates, Jolles and associates). Posterior glenoid wear is a common pattern in OA and not a
contraindication (Walch and associates). However, patients with a brachial plexus root avulsion or
preganglionic injury resulting in a flail arm are not candidates for TSA because of the poor prognosis for
recovery of motor and sensory deficits. Inflammatory arthritis characteristically results in concentric glenoid
wear and not the eccentric posterior erosion seen in OA. This concentric wear results in medialization of the
glenohumeral joint line. This wear pattern can lead to severe erosion, making it difficult to achieve glenoid
resurfacing. Eccentric posterior glenoid wear would lead to posterior humeral head subluxation, which is not
as common in inflammatory arthritis. In addition, inferior humeral head osteophytes are seen in OA. Walch
and associates have developed a classification system describing glenoid wear patterns. During TSA, exposure
to the glenohumeral joint involves subscapularis management, tendon peel, or lesser tuberosity osteotomy. As
a result, early postsurgical rehabilitation limits passive external rotation and active internal rotation, typically
for 6 weeks, to protect the subscapularis repair. The patient in this scenario likely eccentrically contracted his
subscapularis, resulting in repair failure. Upon clinical examination, internal rotation weakness, increased
passive external rotation, and abnormal belly press or lift-off test results can be expected. One study revealed
that rupture of the subscapularis was seen in all anterior dislocations following TSA (Wirth and Rockwood).

Question 83 of 100
A 65-year-old man who underwent an uncomplicated reverse total shoulder arthroplasty (rTSA) to treat rotator
cuff arthropathy 2 years ago has a routine follow-up visit in your clinic. A radiograph at 2-year follow-up is
shown in Figure 1. He denies shoulder pain and dysfunction and constitutional symptoms, and his clinical
examination findings are benign. Based upon the present radiologic evaluation, what is the next most
appropriate step?
A. Revision rTSA
B. Conversion to hemiarthroplasty
C. Continued observation
D. Infection work-up with screening labs and joint aspiration
PREFERRED RESPONSE: C
DISCUSSION
Based upon the patient’s clinical examination and symptoms, continued observation is most appropriate. The
remaining options are not indicated. The radiograph reveals scapular notching, one of the more common
complications specific to rTSA. Notching is caused by repeated contact between the humeral component
and/or humerus and the inferior pillar of the scapular neck. Generation of particulate debris from this
interaction can result in osteolysis with the potential for screw and base plate failure. The overall incidence of
notching has been reported to be between 51% and 96%. This nearly ubiquitous finding has been attributed to
implant positioning, altered glenoid and humeral anatomy, and duration of implantation. Recent studies that
indicate increased lateral offset, increased glenosphere size, and inferior positioning of the base plate may
reduce the incidence of scapular notching.

Question 84 of 100
Figures 1 and 2 are the radiographs of a 69-year-old, left-hand-dominant retired man with left shoulder pain.
The pain has been present for several years. He worked in construction but retired 3 years ago. He now reports
pain interfering with activities around the house but denies recent trauma or prior shoulder surgery. He has
tried nonsteroidal anti-inflammatory drugs, but these do not provide complete relief. He demonstrates pain
and crepitus with active and passive shoulder motion. He can actively forward flex to 100° and external rotate
to 30°. Rotator cuff testing reveals 5/5 strength and he is neurovascularly intact.
Following a successful shoulder arthroplasty, the patient returns for his 1-month follow-up. At today's visit,
his wound appears benign, and he denies drainage or fevers. He reports he was doing well until last week,
when he reached out to close the car door, which resulted in new-onset anterior shoulder pain. His radiograph
from the current visit is shown in Figure 3. What is the most likely cause of this new finding?
A. Supraspinatus tear
B. Subscapularis tear
C. Glenoid component
D. Infection
PREFERRED RESPONSE: B
DISCUSSION
The examination and radiograph findings are consistent with glenohumeral osteoarthritis (OA), which is now
interfering with this patient’s daily activities. Regarding surgical treatment for glenohumeral arthritis, several
studies have shown that TSA is associated with better functional outcomes than hemiarthroplasty. A recent
meta-analysis comparing outcomes between TSA and hemiarthroplasty revealed better function after TSA
regarding pain, University of California-Los Angeles Shoulder Scores, and postsurgical forward elevation at
a minimum of 2 years (Bryant and associates). In addition, Gartsman and associates showed that TSA resulted
in better pain relief, function, strength, and patient satisfaction than hemiarthroplasty at 3-year follow-up. The
patient in this scenario exhibits good rotator cuff strength on examination; therefore, arthroscopy with
debridement or rTSA used in the setting of a massive cuff tear would not be the best answer. Several studies
have similarly shown good results with TSA in the setting of inflammatory and rheumatoid arthritis (Thomas
and associates, Jolles and associates). Posterior glenoid wear is a common pattern in OA and not a
contraindication (Walch and associates). However, patients with a brachial plexus root avulsion or
preganglionic injury resulting in a flail arm are not candidates for TSA because of the poor prognosis for
recovery of motor and sensory deficits. Inflammatory arthritis characteristically results in concentric glenoid
wear and not the eccentric posterior erosion seen in OA. This concentric wear results in medialization of the
glenohumeral joint line. This wear pattern can lead to severe erosion, making it difficult to achieve glenoid
resurfacing. Eccentric posterior glenoid wear would lead to posterior humeral head subluxation, which is not
as common in inflammatory arthritis. In addition, inferior humeral head osteophytes are seen in OA. Walch
and associates have developed a classification system describing glenoid wear patterns. During TSA, exposure
to the glenohumeral joint involves subscapularis management, tendon peel, or lesser tuberosity osteotomy. As
a result, early postsurgical rehabilitation limits passive external rotation and active internal rotation, typically
for 6 weeks, to protect the subscapularis repair. The patient in this scenario likely eccentrically contracted his
subscapularis, resulting in repair failure. Upon clinical examination, internal rotation weakness, increased
passive external rotation, and abnormal belly press or lift-off test results can be expected. One study revealed
that rupture of the subscapularis was seen in all anterior dislocations following TSA (Wirth and Rockwood).
Question 85 of 100
A 17-year-old high school football player sustains a traumatic anterior shoulder dislocation, resulting in a
small bony Bankart lesion and small Hill-Sachs lesion. The patient undergoes an arthroscopic Bankart repair
with incorporation of the bone fragment and returns to play football the following year. He has a recurrent
dislocation at football practice but decides to finish the football season before considering additional treatment.
He sustains nine additional dislocations, with the last dislocation occurring while sleeping.What diagnostic
test is most appropriate when planning revision surgery?
A. CT scan with 3D reconstructions
B. Ultrasonography
C. MRI scan
D. Fluoroscopically-guided arthrogram

PREFERRED RESPONSE: A
DISCUSSION
A failed bony Bankart repair with multiple dislocations can further erode the anteroinferior glenoid, changing
the sagittal morphology of the glenoid into an “inverted pear.” Quantitative bone loss is best evaluated by CT
scan with 3-D reconstructions and subtraction of the humeral head. MRI and ultrasonography can assist in
evaluating soft-tissue injury, but they are not as helpful in determining bone loss compared with a CT scan.
An arthrogram alone is not sufficient to evaluate bone loss. Bone loss >30% necessitates glenoid augmentation
with either a Latarjet procedure or iliac crest bone grafting. A revision arthroscopic or open Bankart repair
with capsular shift or remplissage do not address bone loss. The Latarjet procedure can effectively restore
stability with glenoid bone loss and after failed stabilizing procedures. Patients with pain before surgery are
more likely to have pain after surgery. Age and activity level are lesser influences on satisfaction.

Question 86 of 100
When a patient has recurrent anterior shoulder instability, a bony glenoid reconstructive procedure should be
considered in which clinical setting?
A. Associated humeral avulsion of the glenohumeral ligament (HAGL) lesion
B. Non-engaging Hill-Sachs lesion
C. Glenoid bone loss of at least 25%
D. Anterior labral periosteal sleeve avulsion (ALPSA)

PREFERRED RESPONSE: C
DISCUSSION
HAGL lesions may initially be treated without surgery. Recurrent instability in the setting of a HAGL lesion
may be treated with a soft-tissue repair. A non-engaging or non-tracking Hill-Sachs lesion may be treated with
an anterior soft-tissue (Bankart) repair. A tracking or engaging lesion may be treated with a bony glenoid
procedure or a soft-tissue procedure plus remplissage. An ALPSA lesion may be treated with a soft-tissue
procedure unless it is associated with a glenoid bony defect >25%. A glenoid bony defect >25% is associated
with substantially higher recurrence than defects <20%, and consideration for bony glenoid reconstruction is
advised. Consideration of bone augmentation procedures with less severe glenoid bone loss may be considered
in collision athletes.

Question 87 of 100
A 45-year-old woman with diabetes has a 3-month history of atraumatic left shoulder pain and motion loss.
She previously underwent treatment with nonsteroidal anti-inflammatory medication and a home stretching
program, experiencing minimal relief of her symptoms. Examination reveals loss of passive external rotation,
abduction, and forward elevation without reduction in strength. Radiographs are normal. What is the most
appropriate next step?
A. MRI scan with and without contrast
B. Cortisone injection therapy with continued physical therapy (PT)
C. Closed manipulation under anesthesia
D. Arthroscopic release with manipulation under anesthesia

PREFERRED RESPONSE: B
DISCUSSION
Based upon the duration of symptoms and clinical presentation, this patient would benefit from cortisone
injection therapy and continued PT. Adhesive capsulitis, most commonly an idiopathic process that results in
joint pain and loss of motion from capsular contracture, affects approximately 2% to 5% of the general
population. The process typically affects middle-age women. There are secondary causes such as previous
trauma and fractures, as well as associated medical conditions such as diabetes, stroke, and cardiac and thyroid
disease. Debate remains as to whether a genetic predisposition for the development of adhesive capsulitis
exists, despite the increased frequency noted in twin studies. Although the underlying etiology and
pathophysiology are not well understood, the consensus is that synovial inflammation and capsular fibrosis
result in pain and joint volume loss. It is hypothesized that in patients with diabetes, an increased rate of
glycosylation and cross-linking of the shoulder capsule raises the incidence of frozen shoulder. For this patient,
history reveals a short duration of symptoms that did not improve with nonsurgical modalities. Clinically, the
patient has reduced passive range of motion, particularly with external and internal rotation and forward
elevation. Radiographs are usually obtained to exclude other causes of shoulder pain such as glenohumeral
arthrosis, malignancy, calcific tendonitis, impingement, and acromioclavicular degeneration. If pain and
stiffness persist beyond 6 months, closed manipulation may be an option. Complications associated with this
modality may include humerus fracture, dislocation, hematoma, rotator cuff and labral tears, and brachial
plexus injury. Some surgeons advocate arthroscopic capsular release to allow for examination of concomitant
pathology and controlled release of capsular tissue, with the potential for reduced required force when
performing the manipulation portion of the procedure. This modality may be appropriate after an initial
treatment with PT. Controversy remains as to whether posterior capsular release should be routinely performed
because studies have shown outcomes to be similar with anterior and combined approaches. Therapy should
be initiated early after intervention, with some surgeons advocating admission to the hospital with inpatient
therapy for pain management and compliance.

Question 88 of 100
Figures 1 through 4 are the radiographs and CT scans of a 78-year-old right-hand dominant man with a
recent-onset painful left shoulder and limited range of motion. He was reaching overhead and felt a pop,
which resulted in severe pain and dysfunction. The patient underwent an anatomic total shoulder
arthroplasty 5 years prior for glenohumeral osteoarthritis. A select axial CT image from before the index
surgery is seen in Figure 4. The patient has not had any fevers or systemic symptoms of infection. ESR,
CRP, and CBC levels remain normal. What preoperative factors are most predictive of the complication
experienced by this patient?
A. Patient age at the time of index surgical procedure
B. Presence of a small repairable full-thickness rotator cuff tear
C. Use of pegged all-polyethylene cemented glenoid component
D. Severe baseline glenoid erosion and posterior humeral subluxation
PREFERRED RESPONSE: D
DISCUSSION
This patient has an anatomic total shoulder arthroplasty with a completely dislocated glenoid component. The
glenoid component can be seen in the posterior axillary pouch, as evidenced by the radiographic marker seen
best on the axial cut CT. Preoperative factors that influence the outcome of an anatomic total shoulder
arthroplasty resulting in the need for revision surgery for failed glenoid component include preoperative fixed
posterior humeral head subluxation and moderate to severe eccentric glenoid erosion. Walch and associates
describe three patterns of glenoid component migration including superior tilting, subsidence, and posterior
tilting. Superior tilting was associated with implant position and rotator cuff integrity. Subsidence was
associated with aggressive reaming for correction of glenoid version, which does not maintain the subchondral
bone support. Posterior subluxation was associated with glenoid erosion and posterior humeral head
subluxation.
Question 89 of 100
In rotator cuff tear arthropathy with pseudoparalysis, forward elevation of the humerus away from the
body is prohibited because of
A. deltoid atony.
B. loss of the glenoid concavity.
C. loss of the humeral head depression of the biceps tendon.
D. loss of compressive force on the humeral head.

PREFERRED RESPONSE: D
DISCUSSION
The rotator cuff serves as a humeral head compressor that stabilizes the humeral head in the glenoid
concavity so that the deltoid can convert a vertical force into abduction and forward elevation. The deltoid
functions normally in patients with chronic rotator cuff arthropathy, so no atony is present. Glenoid
concavity can be lost over time, but this is not the primary mechanism for failure of elevation. The biceps
tendon does not serve as a humeral head compressor and does not prevent proximal migration of the
shoulder when it is present.
Question 90 of 100
Figure 1 is the radiograph of a 65-year-old active, right-hand-dominant woman with a 6-month history of
right shoulder pain, motion loss, and progressive weakness after undergoing a hemiarthroplasty to address
osteoarthritis 1 year ago. She denies recent trauma to her right shoulder and denies constitutional
symptoms. Her surgical wound site is benign. She can actively forward flex to 90° degrees and abduct to
60°. Passive forward flexion and abduction are 150° and 90°, respectively.
She completes the necessary testing and wishes to proceed with revision surgery. The most appropriate
surgical option in this scenario involves implant removal and

A. unconstrained total shoulder arthroplasty (TSA).


B. resection arthroplasty.
C. reverse total shoulder arthroplasty (rTSA).
D. hemiarthroplasty.
PREFERRED RESPONSE: C
DISCUSSION
The radiograph reveals a rotator cuff dysfunction secondary to malpositioning of the humeral stem and a
nonanatomic humeral head. Glenohumeral kinematics have been altered, resulting in damage to the
rotator cuff, which in turn has led to impingement with the coracoacromial arch. This single radiograph
reveals excessive humeral head height, “overstuffing” of the joint, and severe narrowing of the
acromiohumeral interval. Osteolysis and implant loosening are not radiographically apparent. An
orthogonal view (axillary lateral) would be necessary to evaluate for shoulder instability. A CT arthrogram
is the most appropriate advanced imaging test in the setting of a retained shoulder arthroplasty to evaluate
the integrity of the rotator cuff. An MRI evaluation would be obfuscated by artifact. Three-phase and
indium-tagged WBC scans may be appropriate in the setting of an occult infection evaluation but not as
a test to evaluate rotator cuff injury. In the absence of infection with rotator cuff compromise, the most
appropriate procedure(s) during revision would involve humeral component explantation and conversion
to rTSA. Revision anatomic hemiarthroplasty may provide pain relief, but function may not appreciably
change because of the unbalanced forced couples of the rotator cuff complex. Placement of a glenoid
component in the setting of an irreparable rotator cuff tear is contraindicated because rapid glenoid
loosening will occur due to eccentric loading during active shoulder motion. Resection arthroplasty should
be reserved for recalcitrant cases of infection, because this procedure does not provide functional
improvement. In the event that frozen section analysis and positive Gram stain results indicate an
infection, the treating surgeon should remove all components, perform a thorough debridement and
irrigation of suspect tissue, implant an antibiotic spacer, and perform a second-stage reconstruction when
deemed appropriate (in light of laboratory studies, repeat shoulder aspiration, frozen section analysis, and
arthroscopic soft-tissue biopsy findings). Irrigation and debridement with primary exchange/conversion
of components remains inferior to 2-stage reconstruction for infection eradication. Resection arthroplasty
remains a salvage procedure for resistant cases that preclude reimplantation and generally is performed
for symptom control and sepsis prevention.

Question 91 of 100
A 23-year-old minor league pitcher describes the insidious onset of posterior shoulder pain during the late
cocking phase of his throwing motion. He has gone 6 weeks without throwing, but symptoms have
returned on return to play. An MR arthrogram of the shoulder reveals fraying of the superior labrum and
proximal biceps, and a partial-thickness articular-sided supraspinatus tear (30% tendon thickness). Figure
1 is a representative coronal MRI slice. Clinical examination demonstrates mild weakness of the
periscapular muscles, mild superior rotator cuff weakness, and negative instability testing. Internal
rotation with the arm in 90° of abduction is 40° in the affected shoulder versus 70° in the contralateral
shoulder. What is the best next step?

A. Intra-articular platelet rich plasma (PRP) injection


B. Therapy regimen focused on shoulder range of motion and strengthening
C. Arthroscopic surgery for rotator cuff and labral debridement
D. Arthroscopic surgery for rotator cuff repair and biceps tenodesis
PREFERRED RESPONSE: B
DISCUSSION
Internal impingement is a condition that affects overhead throwing athletes, as the greater tuberosity and
articular surface of the rotator cuff contact the posterosuperior glenoid during maximal shoulder abduction
and external rotation. The etiology is typically multifactorial, but common contributors include posterior
capsular contracture, scapular dyskinesia, and subtle anterior shoulder laxity. Nonoperative management,
the mainstay of treatment, includes rest, stretching, scapular strengthening, and proprioception. Results
of surgical intervention are variable; therefore, nonoperative measures should be exhausted first. While
PRP is currently being investigated as a biologic augmentation in a number of shoulder pathologies, it is
not considered first-line treatment.
Question 92 of 100
Figure 1 is the radiograph of a 65-year-old active, right-hand-dominant woman with a 6-month history of
right shoulder pain, motion loss, and progressive weakness after undergoing a hemiarthroplasty to address
osteoarthritis 1 year ago. She denies recent trauma to her right shoulder and denies constitutional
symptoms. Her surgical wound site is benign. She can actively forward flex to 90° degrees and abduct to
60°. Passive forward flexion and abduction are 150° and 90°, respectively.
What is the most likely cause of her symptoms?

A. Periprosthetic infection
B. Rotator cuff tear
C. Implant loosening
D. Implant instability
PREFERRED RESPONSE: B
DISCUSSION
The radiograph reveals a rotator cuff dysfunction secondary to malpositioning of the humeral stem and a
nonanatomic humeral head. Glenohumeral kinematics have been altered, resulting in damage to the
rotator cuff, which in turn has led to impingement with the coracoacromial arch. This single radiograph
reveals excessive humeral head height, “overstuffing” of the joint, and severe narrowing of the
acromiohumeral interval. Osteolysis and implant loosening are not radiographically apparent. An
orthogonal view (axillary lateral) would be necessary to evaluate for shoulder instability. A CT arthrogram
is the most appropriate advanced imaging test in the setting of a retained shoulder arthroplasty to evaluate
the integrity of the rotator cuff. An MRI evaluation would be obfuscated by artifact. Three-phase and
indium-tagged WBC scans may be appropriate in the setting of an occult infection evaluation but not as
a test to evaluate rotator cuff injury. In the absence of infection with rotator cuff compromise, the most
appropriate procedure(s) during revision would involve humeral component explantation and conversion
to rTSA. Revision anatomic hemiarthroplasty may provide pain relief, but function may not appreciably
change because of the unbalanced forced couples of the rotator cuff complex. Placement of a glenoid
component in the setting of an irreparable rotator cuff tear is contraindicated because rapid glenoid
loosening will occur due to eccentric loading during active shoulder motion. Resection arthroplasty should
be reserved for recalcitrant cases of infection, because this procedure does not provide functional
improvement. In the event that frozen section analysis and positive Gram stain results indicate an
infection, the treating surgeon should remove all components, perform a thorough debridement and
irrigation of suspect tissue, implant an antibiotic spacer, and perform a second-stage reconstruction when
deemed appropriate (in light of laboratory studies, repeat shoulder aspiration, frozen section analysis, and
arthroscopic soft-tissue biopsy findings). Irrigation and debridement with primary exchange/conversion
of components remains inferior to 2-stage reconstruction for infection eradication. Resection arthroplasty
remains a salvage procedure for resistant cases that preclude reimplantation and generally is performed
for symptom control and sepsis prevention.

Question 93 of 100
A 50-year-old pipefitter falls from a ladder at work and dislocates his non-dominant shoulder. His MRI
scan shows supraspinatus and infraspinatus tears with retraction to the glenoid. He cannot actively raise
his arm away from his side. He denies prior shoulder symptoms before his fall. Three weeks of physical
therapy have failed to improve his function. Which factor has been demonstrated to result in a poor clinical
outcome following surgical intervention?
A. The patient's age
B. he patient's gender
C. Work-related injury
D. Acute nature of the tear

PREFERRED RESPONSE: C
DISCUSSION
Several studies have demonstrated that patients with work-related injuries do not do as well as those
whose injuries are not work-related after repair of the rotator cuff. This patient’s age and gender are not
negative prognostic indicators. The acute nature of the tear does not lead to an inferior outcome.
Question 94 of 100
MRI results are shown in Figure 1 for a 22-year-old, right-hand-dominant collegiate athlete who reports
a 6-month history of progressive weakness in his right arm. He denies any specific traumatic event, has
altered his weight-lifting activities, and has tried over-the-counter ibuprofen without having a benefit. No
appreciable deformity or atrophy is found on examination of the upper extremities. He demonstrates full
active shoulder range of motion, and there is no weakness with abduction in the plane of the scapula.
Belly press test findings are normal, but weakness is seen in external rotation with the arm in adduction.
He does not demonstrate anterior apprehension, and there is no instability with load and shift testing.
Radiographs are unremarkable. What is the best surgical option?
A. Arthroscopic labral debridement and biceps tenodesis
B. Shoulder arthroscopy with undersurface cuff debridement and acromioplasty
C. Cyst decompression at the spinoglenoid notch with possible labral repair
D. Cyst decompression at the suprascapular notch with possible labral repair
PREFERRED RESPONSE: C
DISCUSSION
This patient’s clinical and MRI findings are consistent with a posterior paralabral cyst with compression
of the suprascapular nerve, specifically at the spinoglenoid notch. Compression of the suprascapular nerve
can occur at either the suprascapular or spinoglenoid notch. Compression of the nerve at the suprascapular
notch affects innervation to both the supraspinatus and infraspinatus muscles, resulting in weakness in
both shoulder abduction and external rotation. However, compression at the spinoglenoid notch only
affects innervation to the infraspinatus muscle, resulting in isolated weakness in external
rotation. Compression at the spinoglenoid notch often is seen in overhead athletes, and studies have shown
associated posterior labral tears (Piatt and associates). Several studies have addressed nonsurgical and
surgical treatment options. The treatment decision should focus on the underlying cause (Martin and
associates)—in this patient, the cyst. Nonsurgical treatment in the presence of a known lesion has been
associated with a higher failure rate than addressing the lesion, which can result in functional
improvement (Chen and associates, Cummins and associates). The best response in this scenario is
decompression of the cyst at the spinoglenoid notch with possible labral repair.
Question 95 of 100
Figure 1 is the radiograph of a 47-year-old woman who has pain and difficulty raising her arm after
playing 36 holes of golf in a weekend and now has difficulty sleeping. She denies prior episodes of
shoulder pain. Examination demonstrates guarding with any shoulder motion, tenderness around the
superolateral shoulder, and normal sensory findings.The best initial treatment would entail
A. physical therapy and nonsteroidal anti-inflammatory medications.
B. open biopsy of the lesion for permanent section.
C. manipulation under anesthesia.
D. shoulder arthroscopy.
PREFERRED RESPONSE: A
DISCUSSION
Calcific tendinitis of the shoulder is a deposition of calcium carbonate apatite crystals into the structure
of the rotator cuff tendon. The crystalline form appears to progress throughout the clinical disease process,
demonstrating increasing matured stoichiometric apatite deposition during the resorptive phase. MRI can
be difficult to interpret because the signal of the calcific lesion is frequently similar to that seen in normal
supraspinatus tendon. Plain radiographs remain the gold standard for diagnosis. Ultrasound can be a useful
ancillary study to determine the location and size of the lesion. Primary management of calcific tendinitis
starts with nonsurgical treatment including physiotherapy and injections, if indicated. Mixed results have
been reported with extracorporeal shock wave therapy. Surgical removal with repair of the tendon in
larger lesions remains the definitive treatment when nonsurgical modalities fail. Subacromial
decompression may improve pain relief in patients who require surgery; however, patients with
decompression may take longer to fully recover.
Question 96 of 100
Figures 1 and 2 are the radiographs of a 60-year-old man with gradual onset of right shoulder pain and motion
loss 1 year after undergoing an uncomplicated right total shoulder arthroplasty (TSA) for end-stage
osteoarthritis. He denies trauma to his right shoulder and constitutional symptoms but admits to difficulty
performing activities of daily living. His surgical wound site is benign. He demonstrates active and passive
forward flexion to 90°, abduction to 60°, external rotation to 30°, and internal rotation to the lumbosacral
junction. His rotator cuff strength is graded as normal and symmetrical to his unaffected left shoulder. Based
upon the current evaluation, what is the most appropriate next step?
A. Revision TSA
B. Infection work-up
C. CT scan of the shoulder
D. Observation and physical therapy
PREFERRED RESPONSE: B
DISCUSSION
This patient’s radiographs do not reveal prosthetic loosening, osteolysis, instability, or rotator cuff
deficiency. History and examination findings suggest a possible indolent infectious process, and it is
incumbent upon the treating surgeon to obtain screening laboratory studies (CBC with differential, ESR,
C-reactive protein) and proceed with obtaining cultures (via joint aspiration or arthroscopic soft-tissue
biopsy) to rule out an infectious process. Cultures should be held for 2 weeks to evaluate for the possibility
of a Propionibacterium acnes infection, which is the predominant organism elicited from painful and stiff
TSAs for which revision is required. Immediate revision TSA is not indicated in this scenario because the
components appear well fixed and positioned. Observation and therapy in the setting of deterioration of a
previously well-functioning TSA also are not appropriate.

Question 97 of 100
A 30-year-old man with diabetes sustains an acute posterior dislocation of his right shoulder after a seizure
that required emergency department reduction. You initially treat him with a sling for 4 weeks and then
refer him for outpatient therapy. During his therapy sessions, the patient admits to pain and instability
symptoms during range-of-motion exercises. Repeat examination indicates a positive posterior load-shift
test and apprehension with adduction and internal rotation of the shoulder. His CT and MRI scans are
shown in Figures 1 and 2. What is the most appropriate next step in treatment?
A. Brief period of immobilization and reinitiation of therapy
B. Open posterior capsular shift with labral repair
C. Arthroscopic capsulolabral repair with subscapularis transfer
D. Resurfacing arthroplasty with labral repair
PREFERRED RESPONSE: C
DISCUSSION
Posterior glenohumeral dislocations are much less common than anterior glenohumeral dislocations, with
a prevalence of 1.1 per 100,000 per year. There is a bimodal distribution with a peak in young men (2.4
men to 1 woman) and a second peak in elderly people with a more equivalent gender ratio. Posterior
dislocations most commonly result from trauma, with the remainder of events secondary to seizure
activity. According to Robinson and associates, age <40 years, dislocation during a seizure, and a large
reverse Hill-Sachs lesion were all predictive of recurrent instability. Concomitant injuries associated with
posterior shoulder dislocations include capsulolabral tears, fractures, and rotator cuff tears. Imaging
studies in this patient indicate a reverse Hill-Sachs lesion with a corresponding posterior labral tear.
Because of his persistent mechanical symptoms, continued immobilization and therapy is not appropriate.
An open capsular shift with labral repair alone would not address symptoms related to an engaging reverse
Hill-Sachs lesion and has the added morbidity from the surgical approach. Based upon the patient’s age,
activity level, and percentage of humeral head involvement, a resurfacing arthroplasty is not
recommended. A subscapularis or lesser tuberosity transfer has been used to address symptomatic reverse
Hill-Sachs lesions (20%-40% humeral head involvement) associated with posterior shoulder dislocations.
Modifications of this technique such as arthroscopic transfer of the subscapularis tendon with posterior
capsulorrhaphy have proven beneficial. The indications for the concomitant subscapularis transfer into
the defect (arthroscopic McLaughlin) have not been as well-defined as for patients with a Hill-Sachs
lesion in the setting of recurrent anterior instability.
Question 98 of 100
Figure 1 is the radiograph of a 47-year-old woman who has pain and difficulty raising her arm after
playing 36 holes of golf in a weekend and now has difficulty sleeping. She denies prior episodes of
shoulder pain. Examination demonstrates guarding with any shoulder motion, tenderness around the
superolateral shoulder, and normal sensory findings. An MRI arthrogram scan of her shoulder would most
likely reveal
A. contrast extravasation into the subacromial space.
B. contrast in the glenoid/anterior labral interface.
C. hyperintense heterogenous T2-signal in posterior rotator cuff.
D. hypointense homogenous T1-signal in posterior rotator cuff.
PREFERRED RESPONSE: D
DISCUSSION
Calcific tendinitis of the shoulder is a deposition of calcium carbonate apatite crystals into the structure
of the rotator cuff tendon. The crystalline form appears to progress throughout the clinical disease process,
demonstrating increasing matured stoichiometric apatite deposition during the resorptive phase. MRI can
be difficult to interpret because the signal of the calcific lesion is frequently similar to that seen in normal
supraspinatus tendon. Plain radiographs remain the gold standard for diagnosis. Ultrasonography can be
a useful ancillary study to determine the location and size of the lesion. MRI demonstrates a hypointense
homogenous signal on the T1- and T2 fat suppressed sequences in the region of the calcific deposit. There
may be some increased T2-signal surrounding the lesion indicated localized inflammation. Primary
management of calcific tendinitis starts with nonsurgical treatment including physiotherapy and
injections, if indicated. Mixed results have been reported with extracorporeal shock wave therapy.
Surgical removal with repair of the tendon in larger lesions remains the definitive treatment when
nonsurgical modalities fail. Subacromial decompression may improve pain relief in patients who require
surgery; however, patients with decompression may take longer to fully recover.
Question 99 of 100
In the work-up of a painful shoulder arthroplasty, arthroscopic biopsy specimens that are sent for culture
and discarded after 5 days will most likely yield a false-negative result in the setting of infection by which
organism?
A. Staphylococcus epidermidis
B. Propionibacterium acnes
C. Staphylococcus aureus
D. Streptococcus pneumonia

PREFERRED RESPONSE: B
DISCUSSION
It has been increasingly recognized that P acnes is a common pathogen implicated in infections of the
shoulder. Multiple studies have shown that discarding the cultures prior to 2 weeks will result in a high
rate of false-negative cultures. This is likely because P acnes is a semiobligate anaerobe and grows slowly
in the lab. As such, the clinical presentations are often nonspecific, without overt fever or wound
breakdown, and traditional infection lab values are commonly within normal limits. In addition to S
aureus, P acnes has been cited as the most common etiology for infection following shoulder arthroplasty.
Question 100 of 100
Figure 1 is the radiograph of a 65-year-old active, right-hand-dominant woman with a 6-month history of
right shoulder pain, motion loss, and progressive weakness after undergoing a hemiarthroplasty to address
osteoarthritis 1 year ago. She denies recent trauma to her right shoulder and denies constitutional
symptoms. Her surgical wound site is benign. She can actively forward flex to 90° degrees and abduct to
60°. Passive forward flexion and abduction are 150° and 90°, respectively. Intraoperative frozen section
analysis reveals 10 neutrophils per high-power field and a positive Gram stain result. The most appropriate
next step would consist of implant removal, irrigation and debridement, and

A. resection arthroplasty.
B. placement of rTSA.
C. revision hemiarthroplasty.
D. placement of antibiotic cement spacer.
PREFERRED RESPONSE: D
DISCUSSION
The radiograph reveals a rotator cuff dysfunction secondary to malpositioning of the humeral stem and a
nonanatomic humeral head. Glenohumeral kinematics have been altered, resulting in damage to the
rotator cuff, which in turn has led to impingement with the coracoacromial arch. This single radiograph
reveals excessive humeral head height, “overstuffing” of the joint, and severe narrowing of the
acromiohumeral interval. Osteolysis and implant loosening are not radiographically apparent. An
orthogonal view (axillary lateral) would be necessary to evaluate for shoulder instability. A CT arthrogram
is the most appropriate advanced imaging test in the setting of a retained shoulder arthroplasty to evaluate
the integrity of the rotator cuff. An MRI evaluation would be obfuscated by artifact. Three-phase and
indium-tagged WBC scans may be appropriate in the setting of an occult infection evaluation but not as
a test to evaluate rotator cuff injury. In the absence of infection with rotator cuff compromise, the most
appropriate procedure(s) during revision would involve humeral component explantation and conversion
to rTSA. Revision anatomic hemiarthroplasty may provide pain relief, but function may not appreciably
change because of the unbalanced forced couples of the rotator cuff complex. Placement of a glenoid
component in the setting of an irreparable rotator cuff tear is contraindicated because rapid glenoid
loosening will occur due to eccentric loading during active shoulder motion. Resection arthroplasty should
be reserved for recalcitrant cases of infection, because this procedure does not provide functional
improvement. In the event that frozen section analysis and positive Gram stain results indicate an
infection, the treating surgeon should remove all components, perform a thorough debridement and
irrigation of suspect tissue, implant an antibiotic spacer, and perform a second-stage reconstruction when
deemed appropriate (in light of laboratory studies, repeat shoulder aspiration, frozen section analysis, and
arthroscopic soft-tissue biopsy findings). Irrigation and debridement with primary exchange/conversion
of components remains inferior to 2-stage reconstruction for infection eradication. Resection arthroplasty
remains a salvage procedure for resistant cases that preclude reimplantation and generally is performed
for symptom control and sepsis prevention.

Question 101
Figure 1 is the radiograph of a 47-year-old woman who has pain and difficulty raising her arm after
playing 36 holes of golf in a weekend and now has difficulty sleeping. She denies prior episodes of
shoulder pain. Examination demonstrates guarding with any shoulder motion, tenderness around the
superolateral shoulder, and normal sensory findings. The lesion indicated in the image is comprised of

A. calcium carbonate apatite.


B. hyperproliferative white blood cells.
C. hydroxyapatite crystals.
D. degenerated tenocytes.
PREFERRED RESPONSE: A
DISCUSSION
Calcific tendinitis of the shoulder is a deposition of calcium carbonate apatite crystals into the structure
of the rotator cuff tendon. The crystalline form appears to progress throughout the clinical disease process,
demonstrating increasing matured stoichiometric apatite deposition during the resorptive phase. MRI can
be difficult to interpret because the signal of the calcific lesion is frequently similar to that seen in normal
supraspinatus tendon. Plain radiographs remain the gold standard for diagnosis. Ultrasonography can be
a useful ancillary study to determine the location and size of the lesion. MRI demonstrates a hypointense
homogenous signal on the T1- and T2 fat suppressed sequences in the region of the calcific deposit. There
may be some increased T2-signal surrounding the lesion indicated localized inflammation. Primary
management of calcific tendinitis starts with nonsurgical treatment including physiotherapy and
injections, if indicated. Mixed results have been reported with extracorporeal shock wave therapy.
Surgical removal with repair of the tendon in larger lesions remains the definitive treatment when
nonsurgical modalities fail. Subacromial decompression may improve pain relief in patients who require
surgery; however, patients with decompression may take longer to fully recover.

You might also like